SSC GD 21 Feb 2019 Shift-I Previous Year Paper

SSC GD 21st Feb 2019 Shift-I 

Reasoning 

Instructions 

For the following questions answer them individually 

Q. 1 The statements below are followed by two conclusions labeled I and II. Assuming that the information in the statements is true, even if it appears at variance with generally established facts, decide which conclusion(s) logically and definitely follow(s) from the information given in the statements. 

Statements: 

1) Some stores are rooms. 

2) Some rooms are posters. 

Conclusion: 

I. Some rooms are stores. 

II. Some posters are rooms. 

(A) Only conclusion I follows. 

(B) Both conclusions follow. 

(C) Only conclusion II follows. 

(D) Neither conclusion I nor conclusion II follows. 

Answer: (B) 

Explanation: 

The venn diagram for above statements is : 

Conclusion: 

I. Some rooms are stores = true 

II. Some posters are rooms = true 

Thus, both conclusion follows. 

=> Ans – (B) 

 

Q. 2 Select the option that is related to the third term in the same way as the second term is related to the first term. 

Initial : Final :: Mourn : ? 

(A) Recall 

(B) Recap 

(C) Murmur 

(D) Rejoice 

Answer: (D) 

Explanation: 

The given pairs are antonyms, final is opposite of initial, similarly opposite of mourn is rejoice

=> Ans – (D) 

 

Q. 3 Select the option that will correctly replace the question mark (?) in the series. 

0, 8, 24, 48, ? 

(A) 81 

(B) 87 

(C) 89 

(D) 80 

Answer: (D) 

Explanation: 

Multiples of 8 are added. 

0 + 8 = 8 

8 + 16 = 24 

24 + 24 = 48 

48 + 32 = 80 

=> Ans – (D) 

 

Q. 4 Select the option that is related to the third number in the same way as the second number is related to the first number. 

12 : 180 :: 14 : ? 

(A) 144 

(B) 221 

(C) 169 

(D) 238 

Answer: (D) 

 

Q. 5 Choose the option that most closely resembles the mirror image of the given word if the mirror is placed at the right side. 

Answer: (A) 

Explanation: 

If the mirror is placed at right side, the object at the right will appear at left and vice-versa. Hence the first letter H will appear at right side, and since its symmetrical, its shape won’t be changed. Thus, second option is eliminated. 

Also, in the last two options, the shape of G is changed as such the mirror is placed below it, hence the correct image is shown in first option. 

=> Ans – (A) 

 

Q. 6 The statements below are followed by two conclusions labeled I and II. Assuming that the information in the statements is true, even if it appears at variance with generally established facts, decide which conclusion(s) logically and definitely follow(s) from the information given in the statements. 

Statements: 

1) All customers are clients. 

2) All clients are medicines. 

Conclusion: 

I. Some medicines are customers. 

II. Some medicines are clients. 

(A) Either conclusion I or conclusion II follows. 

(B) Both conclusions follow. 

(C) Only conclusion II follows. 

(D) Only conclusion I follows. 

Answer: (B) 

Explanation: 

The venn diagram for above statements is : 

Conclusion: 

I. Some medicines are customers = true 

II. Some medicines are clients = true 

Thus, both conclusion follow. 

=> Ans – (B) 

 

Q. 7 In certain code, VIROLOGY is written as LOGYVIRO. How will SEMANTIC be written in that code? 

(A) NTICSEMA 

(B) CITNAMES 

(C) NTJCSEMA 

(D) NTICSFMA 

Answer: (A) 

Explanation: 

VIROLOGY is written as LOGYVIRO 

The pattern followed is that the word is divided into two halves, and in the final output, the two halves are interchanged. ≡ 

Eg :- VIROLOGY VIRO and LOGY 

Output : LOGY + VIRO = LOGYVIRO 

Similarly, SEMANTIC : NTICSEMA 

=> Ans – (A) 

 

Q. 8 There is only one runway which will be used by seven flights one by one. P and Q will use the runway neither first nor last. S will use it second after R. M will use it second before O. N will use it after O and before R. Q will use it just before S. Who will use the runway in the middle? 

(A)

(B)

(C)

(D)

Answer: (B) 

Explanation: 

P and Q will not use the runway either first or last. 

S will use it second after R and Q will use it just before S, => Order is RQS 

N will use it after O and before R and M will use it second before O, S will be at end. Final arrangement : 

N will use the runway in the middle. 

=> Ans – (B) 

 

Q. 9 Choose the odd one out of the given options. 

(A) Silicon 

(B) Magnesium 

(C) Tungsten 

(D) Aurum 

Answer: (A) 

 

Q. 10 Choose the odd one out of the given options. 

(A) AGD

(B) IHL 

(C) EKM 

(D) QSC 

Answer: (D) 

Explanation: 

Except for QSC, all the other letters start with a vowel, hence it is the odd one. 

=> Ans – (D) 

 

Q. 11 Eight friends A, B, C, D, E, F, H and G are sitting around a circular table facing each other for a group discussion. A is opposite F and second to the right of E. G is between C and A.H is to the left of E. Who is sitting to the immediate left of A? 

(A) Either G or D 

(B) B

(C) C

(D) Either B or D 

Answer: (D) 

Explanation: 

A is opposite F and second to the right of E, => E is sitting second to the left of A. 

G is between C and A, => G is to the immediate right of A, and C is opposite of E. 

Also, H is to the left of E. Thus, final arrangement : 

∴ Either (B) or (D) is sitting to the immediate left of A. 

=> Ans – (D) 

 

Q. 12 Seven friends O, P, Q, R, S, T and U are watching a cricket match sitting in a stadium.P is sitting at one extreme end. T is sitting between Q and U,and is sitting to the immediate right of S and immediate left of Q. is sitting second to the left of P. Who is sitting between U and P? 

(A)

(B)

(C)

(D)

Answer: (B) 

 

Q. 13 Choose the option that would follow next in the given figure series. 

Answer: (D) 

 

Q. 14 Select the option that is related to the third term in the same wayas the second term is related to the first term. 

AC : CI :: GH : ? 

(A) UY 

(B) UX 

(C) UU 

(D) UV 

Answer: (B) 

Explanation: 

Expression = AC : CI :: GH : ? 

According to the numerical representation of the alphabets, we get A=1, B=2, C=3 and so on. Now, thrice of each letter of first term is equal to the subsequent letters of the second term. 

Eg :- A ≡ 1(×3) = 3 ≡ C 

C ≡ 3(×3) = 9 ≡ I 

and 

G ≡ 7(×3) = 21 ≡ U H ≡ 8(×3) = 24 ≡ X 

Similarly, and 

Thus, GH : UX 

=> Ans – (B) 

 

Q. 15 In a certain code, OPTIMIST is written as 121. How will PESSIMIST be written in that code? 

(A) 129 

(B) 105 

(C) 131 

(D) 127 

Answer: (A) 

Explanation: 

If we add the numbers represented by the English alphabets, i.e. A=1, B=2, C=3, ….., Y=25, Z=26 ≡ 15 + 16 + 20 + 9 + 13 + 9 + 19 + 20 = 121 

OPTIMIST 

≡ 16 + 5 + 19 + 19 + 9 + 13 + 9 + 19 + 20 = 129 

=> PESSIMIST 

=> Ans – (A) 

 

Q. 16 Select the option that will correctly replace the question mark (?) in the series. 

PRL, QPM, RNN, SLO, ? 

(A) TIO 

(B) TKP 

(C) TJP 

(D) TIP 

Answer: (C) 

Explanation: 

Series : PRL, QPM, RNN, SLO, ? 

The pattern followed in each letter of the terms is : 

1st letter : P (+1 letter) = Q (+1 letter) = R (+1 letter) = S (+1 letter) =

2nd letter : R (-2 letters) = P (-2 letters) = N (-2 letters) = L (-2 letters) =

3rd letter : L (+1 letter) = M (+1 letter) = N (+1 letter) = O (+1 letter) =

Thus, missing term : TJP 

=> Ans – (C) 

 

Q. 17 Select the Venn diagram that best represents the given set of classes. 

Animals, Mammals, Deer 

Answer: (D) 

 

Q. 18 Choose the odd number out of the given options. 

(A) 151 

(B) 163 

(C) 157 

(D) 161 

Answer: (D) 

Explanation: 

Among the given numbers, except 161, all are prime numbers. 

161 = 7 × 23 

=> Ans – (D) 

 

Q. 19 Select a figure from amongst the four alternatives, that when placed in the blank space (?) of figure X will complete the pattern. (Rotation is not allowed). 

Answer: (C) 

 

Q. 20 Find out the two signs to be interchanged to make the following equation correct. 

18 + 3 × 27 − 9 ÷ 3 = 78 

(A) + and × 

(B) + and ÷ 

(C) − and ÷ 

(D) + and − 

Answer: (A) 

Q. 21 Select the option that will correctly replace the question mark (?) in the series. 

12, 22, 42, 82, ? 

(A) 172 

(B) 182 

(C) 152 

(D) 162 

Answer: (D) 

Explanation: 

The pattern followed is : 

12 + 10 = 22 

22 + 20 = 42 

42 + 40 = 82 

82 + 80 = 162 

=> Ans – (D) 

 

Q. 22 Select the option that will correctly replace the question mark (?) in the given pattern. 

(A)

(B) 24 

(C) 36 

(D)

Answer: (A) 

 

Q. 23 A square transparent sheet with a pattern is given. How will the pattern appear when the transparent sheet is folded along the dotted line? 

Answer: (A) 

 

Q. 24 Select the option that is related to the third term in the same way as the second term is related to the first term. 

Blend : Mix :: Request : ? 

(A) Deny 

(B) Noting 

(C) Solicit 

(D) Accept 

Answer: (C) 

Explanation: 

The first two words are synonyms of each other, similarly request and solicit have similar meanings. 

=> Ans – (C) 

 

Q. 25 Select a figure from amongst the four alternatives, that when placed in the blank space (?) of figure X will complete the pattern. (Rotation is not allowed). 

Answer: (D) 

General knowledge 

Instructions 

For the following questions answer them individually 

Q. 26 Which of the following statements about the Indus Valley Civilization is NOT true? 

(A) It was an urban civilisation where people lived in well planned towns 

(B) Animal meat was the staple food 

(C) It flourished around 2500 BCin the present day Pakistan and Western India 

(D) The houses were made of baked bricks 

Answer: (B) 

 

Q. 27 Dry Ice is the solid form of: 

(A) methane 

(B) carbon dioxide 

(C) oxygen 

(D) ozone 

Answer: (B) 

 

Q. 28 Timur invaded India during the reign of: 

(A) Akbar 

(B) Alauddin Khilji 

(C) Feroz Shah Tughlaq 

(D) Nasir-ud-Din Mahmud Shah Tughluq 

Answer: (D) 

 

Q. 29 Who wrote the book ‘War and Peace’? 

(A) Viadimir Lenin 

(B) Leo Tolstoy 

(C) Karl Marx 

(D) Anton Chekhov 

Answer: (B) 

 

Q. 30 India has won 8 Olympic gold medals for field hockey. When was the last time India won a gold medal for hockey in Olympics? 

(A) 1976 

(B) 1984 

(C) 1988 

(D) 1980 

Answer: (D) 

 

Q. 31 Which of the following statements is NOT true about angiosperms? 

(A) They have stems, leaves and roots 

(B) Conifers, Cycads and Ginkgo are examples of angiosperms 

(C) They have to undergo pollination before they can reproduce 

(D) The flowers carry the reproductive organs 

Answer: (B) 

 

Q. 32 The economic survey is usually presented ……… before the Union Budget. 

(A) 1 day 

(B) 15 days 

(C) 7 days 

(D) 2 days 

Answer: (A) 

 

Q. 33 Gita Gopinath became the first woman to occupy the position of Chief Economist of: 

(A) World Trade Organisation 

(B) UNESCO 

(C) World Bank 

(D) International Monetary Fund 

Answer: (D) 

 

Q. 34 The holy place of the Jains, Shravanabelagola is situated in which district of Karnataka? 

(A) ShimogaDistrict 

(B) Bellary District 

(C) Mysore District 

(D) Hassan District 

Answer: (D) 

 

Q. 35 The Government of India releases MSP of agricultural produce from time to time. What is the full form of MSP? 

(A) Minimum Selling Price 

(B) Minimum Support Price 

(C) Maximum Selling Price 

(D) Maximum Support Price 

Answer: (B) 

 

Q. 36 Golconda Fort was built in the 13th century by the …………. Dynasty. 

(A) Kakatiya 

(B) Chola 

(C) Periyar 

(D) Chalukya 

Answer: (A) 

 

Q. 37 In the context of climate. what is the full form of ITCZ? 

(A) Indian Trough Conversation Zone 

(B) Inter Tropical Convergence Zone 

(C) Intra Temperate Control Zone 

(D) Islandic Tropical Climate Zone 

Answer: (B) 

 

Q. 38 The Indian athlete Dutee Chand is associated to which of the following sporting events? 

(A) Shotput 

(B) Javelin 

(C) Jumping 

(D) Sprinting 

Answer: (D) 

 

Q. 39 A special property of acids and bases is their ability to ……… each other’s properties. 

(A) Crystalize 

(B) Neutralize 

(C) Amortize 

(D) Naturalize 

Answer: (B) 

 

Q. 40 According to the report published by the World Health Organization (WHO)in 2017, approximately how many people in the world do NOT have access to safe drinking water at home? 

(A) 4.2 billion 

(B) 900 million 

(C) 500 million 

(D) 2.1 billion 

Answer: (D) 

 

Q. 41 ‘Kuchipudi’ derives its roots from: 

(A) Kerala 

(B) Andhra Pradesh 

(C) Puducherry 

(D) Tamil Nadu 

Answer: (B) 

 

Q. 42 Swachh Bharat Abhiyan was launched by the government of India on the birth anniversary of: 

(A) Mahatma Gandhi 

(B) Vallabhbhai Patel 

(C) Jawaharlal Nehru 

(D) Atal Bihari Vajpayee 

Answer: (A) 

 

Q. 43 The Constitution of India came into force on 26 January 1950. Which act or governing document did it replace? 

(A) Government of India Act, 1858 

(B) Government of India Act, 1935 

(C) Indian Councils Act, 1892 

(D) Regulating Act, 1773 

Answer: (B) 

 

Q. 44 The parliament has powers to amend the provisions of the Constitution of India. Which article of the constitution confers this power on the parliament? 

(A) Article 1 

(B) Article 256 

(C) Article 55 

(D) Article 368 

Answer: (D) 

 

Q. 45 The first major nation wide Satyagraha launched by Mahatma Gandhi was against: 

(A) Rowlatt Act 

(B) Partition of Bengal 

(C) Tax on Khadi products 

(D) Salt tax 

Answer: (A) 

 

Q. 46 Which of the following is NOT true about the ‘Protection of Women from Domestic Violence Act, 2005’? 

(A) Injury can be physical, emotional or economic. 

(B) It also applies to violence at workplace. 

(C) It applies in case of injury or harm or threat of injury or harm by the adult male member of the house hold. 

(D) The term ‘Domestic’ includes all women who liveor have lived in a shared household with the accused male member. 

Answer: (B) 

 

Q. 47 The host country for 2022 FIFA World Cup is: 

(A) Japan 

(B) Germany 

(C) France 

(D) Qatar 

Answer: (D) 

 

Q. 48 ‘Slash and burn’ type of farming is also known as: 

(A) shifting farming 

(B) intensive subsistence farming 

(C) extensive farming 

(D) commercial farming 

Answer: (A) 

 

Q. 49 What is the monthly salary of the President of India as per 2018 Budget? 

(A) 4 lakhs 

(B) 2 lakhs 

(C) 5 lakhs 

(D) 3 lakhs 

Answer: (C) 

 

Q. 50 Which of the following is NOT a method of soil conservation? 

(A) Intercropping 

(B) Mulching 

(C) Contour Ploughing 

(D) Weathering 

Answer: (D) 

Quant 

Instructions 

For the following questions answer them individually 

Q. 51 A sector is cut from a circle of diameter 42 cm. If the angle of the sector is 150then what is its area in cm2. (Take π = 22/7) 

(A) 584.8 

(B) 577.5 

(C) 564.6 

(D) 580.4 

Answer: (B) 

 

Q. 52 The present ages of A and are in the ratio 15 : 8. After 8 years their ages will be in the ratio 17 : 10. What will be the ratio of ages of A and B after 10 years from now? 

(A) 10 : 7 

(B) 8 : 5 

(C) 5 : 3 

(D) 7 : 4 

Answer: (C) 

 

Q. 53 A circus tent is cylindrical to a height of 3 meters and conical above it. If the radius of the base is 52.5 m and the slant height of the cone is 52 m,then the total area of the canvas required to make it is: 

(A) 3048π 

(B) 3045π 

(C) 2730π 

(D) 2842π 

Answer: (B) 

 

Q. 54 The compound interest on a sum of ₹ 24500 at 10% p.a for 2⅖ years interest compounded yearly is: 

(A) ₹ 6338.40 

(B) ₹ 6342 

(C) ₹ 6330.80 

(D) ₹ 6340 

Answer: (C) 

Instructions 

Study the following table, which shows the number of units of a product manufactured and sold by a company in five years, and answer questions. 

 

Q. 55 What percent of total units of the product manufactured in 2013 and 2015 is the total units unsold during the five years? (correct to one decimal place) 

(A) 30.8 

(B) 30.3 

(C) 29.6 

(D) 29.4 

Answer: (B) 

Q. 56 The ratio of the total number of units sold in 2014 and 2015 to the number of units manufactured in 2017 is: 

(A) 17:12 

(B) 19:14 

(C) 9:8 

(D) 14:17 

Answer: (A) 

Instructions 

For the following questions answer them individually 

 

Q. 57 The value of 

1.6 ÷ 1.6 of 1.6 + 0.6 of 1.4 ÷ 0.5 − 0.001 × 1.5 ÷ 0.075 

lies between: 

(A) 3.4 and 3.6 

(B) 3.6 and 3.8 

(C) 3.2 and 3.4 

(D) 3 and 3.2 

Answer: (C) 

 

Q. 58 The average of 10 numbers is 41. The average of first 3 numbers is 39.4 and the average of last 4 numbers is 43.5. 4th number is 2 more than the 5th number and 0.2 more than the 6th number. What is the average of the 4th and 6th numbers .

(A) 38.7 

(B) 38.9 

(C) 39.6 

(D) 39.9 

Answer: (D) 

 

Q. 59 The average age of 24 students in a class is 15.5 years. The age of their teacher is 28 years more than the average of all twenty five. What is the age of the teacher in years? 

(A) 40⅓ 

(B) 41⅔ 

(C) 44⅔ 

(D) 42⅓ 

Answer: (C) 

 

Q. 60 If 12, x, 8 and 14 are in proportion, then what is the mean proportional between (x – 12) and (x + 4) ? 

(A) 12 

(B) 11 

(C) 16 

(D) 15 

Answer: (D) 

 

Q. 61 A 280 m long train is running at 108 km/hr. In how much time will it cross a bridge of length 170 m ? 

(A) 10 sec 

(B) 15 sec 

(C) 16 sec 

(D) 12 sec 

Answer: (B) 

 

Q. 62 A shopkeepers made a profit of 16% on his goods .If the price at which he buys decreases by 20% while he reduces his selling price by 23%, what is now his percentage of profit ? (correct to one decimal place) 

(A) 11.6 

(B) 15.4 

(C) 12.9 

(D) 14.6 

Answer: (A) 

 

Q. 63 The largest sphere is carved out of a cube of side 7 cm. What is the volume of the sphere in cm3 (Take π = 22/7

(A) 179⅔ 

(B) 182⅓ 

(C) 179⅓ 

(D) 182⅔ 

Answer: (A) 

 

Q. 64 Two vessels A and B contain solution of acid and water. In A and B the ratio of acid and water are 7 : 3 and 4 : 1, respectively. They are mixed in the ratio 1 : 2. What is the ratio of acid and water in the resulting solution? 

(A) 15 : 8 

(B) 16 : 7 

(C) 23 : 7 

(D) 25 : 9 

Answer: (C) 

 

Q. 65 Let x be the least multiple of 29 which when divided by 20, 21, 22, 24 and 28 then the remainders are 13, 14, 15, 17 and 21 respectively. What is the sum of digits of x? 

(A) 18 

(B) 22 

(C) 19 

(D) 23 

Answer: (D) 

 

Q. 66 At what percentage above the cost price must an article be marked so as to gain 38% after allowing a customer a discount of 8% on the marked price? 

(A) 48 

(B) 46 

(C) 50 

(D) 45 

Answer: (C) 

 

Q. 67 The numbers 6, 8, 11, 12, 2x – 8, 2x + 10, 35, 41, 42, 50 are written in ascending order. If their median is 25, then what is the mean of the numbers? 

(A) 24.8 

(B) 24.6 

(C) 25.5 

(D) 25.2 

Answer: (C) 

 

Q. 68 A takes 6 hours more than B to cover a distance of 60 km. But if A doubles his speed, he takes 3 hours less than B to cover the same distance. The speed (in km/hr) of A is: 

(A) 3⅓ 

(B) 4⅔ 

(C)

(D)

Answer: (A) 

 

Q. 69 A sum of ₹ 1518 is divided among A, B, and C, such that A receives 20% more than B and B receives 20% less than C. What is the share of C in the sum? 

(A) ₹ 528 

(B) ₹ 440 

(C) ₹ 550 

(D) ₹ 572 

Answer: (C) 

 

Q. 70 Study the following table, which shows the number of units of a product manufactured and sold by a company in five years, and answer questions. 

In which year the number of units of the product sold is 19000 more than the average number of units manufactured per year? 

(A) 2016 

(B) 2015 

(C) 2017 

(D) 2014 

Answer: (C) 

 

Q. 71 The simple interest on a certain sum is 1119% of the sum and the number of years is equal to the rate percent per annum. What will be the simple interest on a sum of ₹ 12600 at the same rate for 2⅘ years? 

(A) ₹1216 

(B) ₹1224 

(C) ₹1176 

(D) ₹1164 

Answer: (C) 

 

Q. 72 A can complete ⅔ of a work in 8 days. B can complete ⅗ of the same work in 12 days and C can complete 4/9 of the same work in 8 days. A and B worked together for 5 days. How much time(in days) will C alone take to complete the remaining work? 

(A)

(B) 7½ 

(C)

(D)

Answer: (A) 

 

Q. 73 The ratios of efficiencies of A and B of doing a certain work is 5 : 8. Working together they can complete the work in 20 days, A alone will complete 25% of that work in: 

(A) 14 days 

(B) 12 days 

(C) 11 days 

(D) 13 days 

Answer: (D) 

 

Q. 74 If 20% more would be gained by selling an article for ₹ 93 than selling it for ₹ 85, then the cost price of the article is: 

(A) ₹64 

(B) ₹40 

(C) ₹56 

(D) ₹60 

Answer: (B) 

 

Q. 75 The value of 

35/6 ፥7/2 ✖21/10 +3/5 of 15/2 ፥2/3 – 2/3 ፥8/15 ✖6/5

(A) 8¾ 

(B) 2⅗ 

(C) 8⅗ 

(D) 1¾ 

Answer: (A) 

English 

Instructions 

For the following questions answer them individually 

Q. 76 Select the antonym of the given word. 

GRANT 

(A) Refuse 

(B) Approve 

(C) Reject 

(D) Sanction 

Answer: (A) 

 

Q. 77 Select the most appropriate option to substitute the underlined segment in the given sentence. If there is no need to substitute it, select No improvement. 

(A) team of doctors is focusing over treatment of cancer at an early age. 

(A) is focusing on to treatment of cancer 

(B) is focusing on treatment of cancer 

(C) No improvement 

(D) is focusing in treatment of cancer 

Answer: (B) 

 

Q. 78 From the given options, identify the segment in the sentence which contains the grammatical error. As a painter he are good if not better than Suresh. 

(A) he are good 

(B) than Suresh 

(C) if not better 

(D) As a painter 

Answer: (A) 

 

Q. 79 Select the most appropriate option to substitute the underlined segment in the given sentence.If there is no need to substitute it, select No improvement. 

He have been working on and off for two years to complete a book. 

(A) has work 

(B) have worked 

(C) No improvement 

(D) has been working 

Answer: (D) 

 

Q. 80 Select the word which means the same as the group of words given. 

One who makes an eloquent public speech 

(A) Leader 

(B) Debater 

(C) Speaker 

(D) Orator 

Answer: (D) 

 

Q. 81 Select the most appropriate option to substitute the underlined segment in the given sentence. If there is no need to substitute it, select No improvement. 

Would you mind help me with these questions? 

(A) Would you mind helping 

(B) Would you mind for helping 

(C) No improvement 

(D) Would you mind to help 

Answer: (A) 

 

Q. 82 Select the word which means the same as the group of words given. 

The absence of government in a country 

(A) Bureaucracy 

(B) Anarchy 

(C) Autocracy 

(D) Democracy 

Answer: (B) 

 

Q. 83 Select the option that means the same as the given idiom. 

In black and white 

(A) In writing 

(B) In colour 

(C) Scanned 

(D) Photocopied 

Answer: (A) 

Instructions 

In the following passage some words have been deleted. Fill in the blanks with the help ofthe alternatives given. 

Passage: 

In 1893, Lokmanya Tilak converted the Ganapati festival …………(1) a national celebration. He campaigned for ………….(2) wide celebration of this public festival throughout Maharashtra. It was …………(3) this festival that he …………..(4) mobilise public support for the nationalist …………(5) 

 

Q. 84 Select the most appropriate option that will fill in the blank number 1. 

(A) with 

(B) into 

(C) for 

(D) to 

Answer: (B) 

 

Q. 85 Select the most appropriate option that will fill in the blank number 2. 

(A)

(B) some 

(C) the 

(D) one 

Answer: (C) 

 

Q. 86 Select the most appropriate option that will fill in the blank number 3. 

(A) with 

(B) through 

(C) without 

(D) throughout 

Answer: (B) 

 

Q. 87 Select the most appropriate option that will fill in the blank number 4. 

(A) may 

(B) can 

(C) will 

(D) could 

Answer: (D) 

 

Q. 88 Select the most appropriate option that will fill in the blank number 5. 

(A) movement 

(B) term 

(C) season 

(D) festival 

Answer: (A) 

Instructions 

For the following questions answer them individually 

 

Q. 89 Select the most appropriate option to fill in the blank. 

‘Discovery of India’ ……… by Jawaharlal Nehru. 

(A) have been written 

(B) has written 

(C) has been written 

(D) have written 

Answer: (C) 

 

Q. 90 Select the correctly spelt word. 

(A) Aquaintance 

(B) Acquantance 

(C) Acquaintance 

(D) Acquaintence 

Answer: (C) 

 

Q. 91 From the given options, identify the segment in the sentence which contains the grammatical error. When I asked for directions, he gave me many wrong informations. 

(A) When I asked 

(B) for directions 

(C) wrong informations 

(D) gave me many 

Answer: (C) 

 

Q. 92 Select the synonym of the given word. 

ZENITH 

(A) Peak 

(B) Border 

(C) Brink 

(D) Edge 

Answer: (A) 

 

Q. 93 Select the option that means the same as the given idiom. 

Hit the nail on the head 

(A) Accurately right 

(B) Hit the nail exactly 

(C) Use hammer and nail 

(D) Use some tools 

Answer: (A) 

 

Q. 94 Select the synonym of the given word. 

RESISTANCE 

(A) Approval 

(B) Recognition 

(C) Refusal 

(D) Enthusiasm 

Answer: (C) 

 

Q. 95 Select the antonym of the given word. 

HUMILITY 

(A) Cruelty 

(B) Shyness 

(C) Obedience 

(D) Modesty 

Answer: (D) 

 

Q. 96 Select the most appropriate option to fill in the blank. 

One should be careful to check the grease and oil in ……….. car occasionally for it’s smooth functioning 

(A) ones 

(B) their 

(C) our 

(D) one’s 

Answer: (D) 

 

Q. 97 From the given options, identify the segment in the sentence which contains the grammatical error. Is the older person always more wiser than the younger one? 

(A) Is the 

(B) more wiser 

(C) older person always 

(D) than the younger one? 

Answer: (B) 

 

Q. 98 Select the most appropriate word to fill in the blank. 

Many tourists to India visit the beautiful valley of Kashmir ………. in spring and summer. 

(A) generally 

(B) rarely 

(C) especially 

(D) frequently 

Answer: (C) 

 

Q. 99 Select the most appropriate word to fill in the blank. 

Our society is ………. by the evil of corruption; the integrity of even senior officials is disputed. 

(A) indicated 

(B) endangered 

(C) specified 

(D) identified 

Answer: (B) 

 

Q. 100 Select the correctly spelt word. 

(A) Millennium 

(B) Milennium 

(C) Millenniam 

(D) Millenium 

Answer: (A) 

SSC GD 19 Feb 2019 Shift-III Previous Year Paper

SSC GD 19th Feb 2019 Shift-III

Reasoning 

Instructions 

For the following questions answer them individually 

Q. 1 Select the correct option that will fill in the blank and complete the series. 

5, …….., 10, 19, 35, 60 

(A)

(B)

(C)

(D)

Answer: (A) 

Explanation: 

5 + 12 = 6 

6 + 22 = 10 

10 + 32 = 19 

19 + 42 = 35 

35 + 52 = 60 

Hence missing number series will be 6. 

5, 6, 10, 19, 35, 60…. 

 

Q. 2 The statements below are followed by two conclusions labeled I and II. Assuming that the information in the statements is true, even if it appears to be at variance with generally established facts, decide which conclusion(s) logically and definitely follow(s) from the information given in the statements. 

Statements: 

1. Some animals are tigers. 

2. All tigers are vertebrates. 

Conclusions: 

I. All vertebrates are tigers. 

II. Some vertebrates are animals. 

(A) Both conclusion I & II follow 

(B) Only conclusion II follows 

(C) Only conclusion I follows 

(D) Neither conclusion I nor II follows 

Answer: (B) 

Explanation: 

Conclusions: 

I. All vertebrates are tigers- False 

II. Some vertebrates are animals- True. 

Only Conclusion II follows. 

 

Q. 3 The numbers in the following set are related in a certain way. Choose the set that is similar to the following set: 

{128, 8, 32} 

(A) {196, 7, 14} 

(B) {122, 10, 30} 

(C) {144, 36, 72} 

(D) {64, 4, 16} 

Answer: (D) 

Explanation: 

128+32 = 180 

180 ÷ 20 = 4 

Similarly in option D, 64+16 = 80 

80 ÷ 20 = 4 

Hence Option (D) is correct option. 

 

Q. 4 Choose the alternative which most closely resembles the mirror image of the given word when mirror is placed at right side. 

Answer: (A) 

 

Q. 5 Which number will follow next in the given number series? 

15, 12, 19, 14, 25, 16, ? 

(A) 29 

(B) 31 

(C) 35 

(D) 33 

Answer: (D) 

 

Q. 6 Select the figure which when placed in the blank space of the figure marked ‘X’ would complete the pattern. 

Answer: (B) 

 

Q. 7 R is taller than T. Both P and Q are taller than S, whose height is between P and T. If is taller than T, who is the shortest? 

(A)

(B)

(C)

(D)

Answer: (D) 

Explanation: 

missing value in question 

IF ____ is taller than T, who is the 

 

Q. 8 Choose the option that would follow next in the given figure series. 

Answer: (D) 

 

Q. 9 Pick the odd pair out. 

(A) 17- 49 

(B) 5-13 

(C) 9-25 

(D) 15- 41 

Answer: (D) 

 

Q. 10 Six persons A, B, C, D, E and F are sitting around a round table facing the center. B is seated to the left of D. E is seated to the right of C. A is facing F, who is seated to the left of C. Who is seated to the left of A? 

(A)

(B)

(C)

(D)

Answer: (A) 

 

Q. 11 Select the correct option that will fill in the blank and complete the series. 

dmt, fkv, ihy, mdc, …….. 

(A) ryh 

(B) rzh 

(C) qzl 

(D) rxi 

Answer: (A) 

Explanation: 

Take all the first digit of all the words 

d+2=f , f+3=i , i+4=m, Hence m+5=R 

Take all the second digit of all the words 

m-2=k , k-3=h , h-4=d , Hence d-5=Y 

Take all the third and last digit of all the words 

t+2=v, v+3=y, y+4=c, Hence c+5=H 

Let’s consolidate the whole word = RYH. 

Hence Sequence will be dmt, fkv, ihy, mdc, ryh.. 

 

Q.12 AGENDA is related to MEETING in the same way as PROGRAMME is related to: 

(A) FUNCTION 

(B) INAUGURATION 

(C) SPEECH 

(D) ORGANISATION 

Answer: (A) 

Explanation: 

Agenda is same meaning word for Meeting. 

Similarly, Programme is same meaning word for Function 

 

Q. 13 Select the option that is related to the third term in the same way as the second term is related to the first term. 

Temple : Worship :: Court : ? 

(A) Argument 

(B) Judge 

(C) Jurisdiction 

(D) Lawyer 

Answer: (A) 

Explanation: 

In Temple we do worship. Similarly, in Court we do argument. 

 

Q. 14 Four numbers have been given out of which three are alike in some manner, while one is different. Choose the odd one. 

(A) 125 

(B) 729 

(C) 1321 

(D) 343 

Answer: (C) 

 

Q. 15 Which of the following Venn diagrams correctly represents the relationships among the classes: 

Products, Factory, Machines 

Answer: (A) 

 

Q. 16 If in a code language ADVENTURES is coded as VDANEUTSER, then SYSTEMATIC would be coded as: 

(A) CITETAMSYS 

(B) SYSETAMCIT 

(C) SYSMETITAC 

(D) YSTSMETACI 

Answer: (B) 

 

Q. 17 If in a code language BADE = 1254 and HATE = 18520, then HEALER = ? 

(A) 58121185 

(B) 18512158 

(C) 15818512 

(D) 58112158 

Answer: (A) 

 

Q. 18 Six girls are sitting in a row facing West. Kim is to the left of Sima and Sony is between Piku and Ria. There are two people seated between Piku and Sima, who is separated from Neela by one person.If Piku is seated at one end of the row, who is at the other end? 

(A) Sima 

(B) Kim 

(C) Ria 

(D) Neela 

Answer: (D) 

 

Q. 19 Which one of the following four-letter clusters does NOT belong to the group? 

(A) PVTR 

(B) CIGE 

(C) WZXY 

(D) KQOM 

Answer: (C) 

Explanation: 

PVTR = P+6=V 

CIGE = C+6=I 

WZXY = W+2=Y 

KQOM = K+6=Q 

Hence WZXY is odd as from first letter to second letter W-to-Z is only gapping of 2 Alphabets. 

 

Q. 20 Choose the option in which the figure marked ‘x’ is embedded. 

Answer: (C) 

 

Q. 21 The statements below are followed by two conclusions labeled I and II. Assuming that the information in the statements is true, even if it appears to be at variance with generally established facts, decide which conclusion(s) logically and definitely follow(s) from the information given in the statements. 

Statements: 

1. All roads are streets. 

2. Some streets are avenues. 

Conclusions: 

I. Some roads are avenues. 

II. All avenues are roads. 

(A) Both conclusions I and II follow 

(B) Neither conclusion I nor II follows 

(C) Only conclusion I follows 

(D) Only conclusion II follows 

Answer: (B) 

 

Q. 22 Select the option that is related to the third term in the same way as the second term is related to the first term. 

BYWD : DWUF :: FUSH : ? 

(A) JQOL 

(B) HSJQ 

(C) HTRJ 

(D) HSQJ 

Answer: (D) 

Q. 23 Pick the odd one out. 

(A) Plate 

(B) Ring 

(C) Bangle 

(D) Tyre 

Answer: (A) 

Explanation: 

All the shapes are round. But Plates has no hole inside circle.. But Ring,Tyre,Bangle has hole inside circle. Hence Plate is odd option. 

 

Q. 24 Which two signs should be interchanged to make the following equation correct? 

6 × 2 + 8 − 16 ÷ 4 = 18 

(A) + and ÷ 

(B) + and × 

(C) × and ÷ 

(D) − and ÷ 

Answer: (B) 

 

Q. 25 How would the pattern given on the square transparent sheet marked ‘X’ look when the sheet is folded along the dotted line? 

Answer: (C) 

General knowledge 

Instructions 

For the following questions answer them individually 

Q. 26 Which among the following is the smallest mountain range in the world? 

(A) Ural 

(B) Sutter Buttes 

(C) Alps 

(D) Sierra Nevada 

Answer: (B) 

 

Q. 27 Who is the founder of ‘Sulabh Sanitation and Social Reform Movement’? 

(A) Aruna Roy 

(B) Medha Patkar 

(C) Dr Bindeshwar Pathak 

(D) Baba Amte 

Answer: (C) 

 

Q. 28 Which of the following is regarded as the shortest river of India? 

(A) Narmada 

(B) Arvari 

(C) Kaveri 

(D) Godavari 

Answer: (B) 

 

Q. 29 ‘Jhumur’ a traditional dance form, is performed in which of the following states? 

(A) Odisha 

(B) Madhya Pradesh 

(C) Bengal 

(D) Assam 

Answer: (D) 

 

Q. 30 Aravind Adiga got the prestigious Man Booker Prize for which book? 

(A) Selection Day 

(B) The Kite Runner 

(C) The White Tiger 

(D) God of Small Things 

Answer: (C) 

 

Q. 31 ‘The Mousetrap’ was a famous play written by: 

(A) Agatha Christie 

(B) Caryl Churchill 

(C) William Shakespeare 

(D) George Bernard Shaw 

Answer: (A) 

 

Q. 32 India recognizes the Right to ………… as being part of Right to Life (Article 21). 

(A) Medication 

(B) Acceptance 

(C) Fitness 

(D) Water 

Answer: (D) 

 

Q. 33 Eminent economist and columnist ……….. has resigned as part time member of Economic Advisory Council to the Prime Minister Narendra Modi. 

(A) Surjit Bhalla 

(B) Arvind Subramanian 

(C) Raghuram Rajan 

(D) Jagdish Bhagwati 

Answer: (A) 

 

Q. 34 Which Indian state has the largest area under forest cover? 

(A) Kerala 

(B) Chhattisgarh 

(C) Arunachal Pradesh 

(D) Madhya Pradesh 

Answer: (D) 

 

Q. 35 In which year was the Right to Education Act enacted in India? 

(A) 2011 

(B) 2004 

(C) 2009 

(D) 2006 

Answer: (C) 

 

Q.36 Which country among the following options has the shortest constitution in the world? 

(A) Norway 

(B) United Kingdom 

(C) United States of America 

(D) India 

Answer: (C) 

 

Q. 37 Which city among the following is the most densely populated city, as of 2018? 

(A) Shanghai 

(B) Tokyo 

(C) Paris 

(D) Istanbul 

Answer: (B) 

 

Q. 38 What was the event that fundamentally changed Sardar Vallabhbhai Patel’s life and drove him towards politics? 

(A) Bardoli Satyagraha (year 1928) 

(B) The Gandhi Irwin Pact (year 1931) 

(C) Meeting with Mahatma Gandhi in Godhra (year 1917) 

(D) Non-Cooperation Movement (year 1922) 

Answer: (C) 

 

Q. 39 During whose rule was the historical Nalanda University established? 

(A) Bindusara 

(B) Dasratha 

(C) Chandragupta 

(D) Kumaragupta 

Answer: (D) 

 

Q. 40 In which year did Ms. Sushmita Sen became the first Indian woman to win the ‘Miss Universe’ title? 

(A) 1996 

(B) 1994 

(C) 1998 

(D) 1991 

Answer: (B) 

 

Q. 41 Neutral, expansionary, and contractionary strategies are related to which policy of the government? 

(A) Monetary policy 

(B) Banking policy 

(C) Budgetary policy 

(D) Fiscal policy 

Answer: (D) 

 

Q. 42 The famous ‘Ratnagiri Monastery’ is located in which Indian state? 

(A) Karnataka 

(B) Gujarat 

(C) Orissa 

(D) Jharkhand 

Answer: (C) 

 

Q. 43 Which of the following is a nocturnal animal? 

(A) Hamster 

(B) Bat 

(C) Porcupine 

(D) Murids 

Answer: (B) 

 

Q. 44 ‘Hemis Festival’ is celebrated in which Indian state? 

(A) Jammu and Kashmir 

(B) Karnataka 

(C) Tamil Nadu 

(D) Punjab 

Answer: (A) 

 

Q. 45 The ancient cities of Harappa and Mohenjo-Daro emerged along the banks of which river? 

(A) Baghmati River 

(B) Indus River 

(C) Brahmaputra River 

(D) Ganga River 

Answer: (B) 

 

Q. 46 Who has won Sayed Modi India Grand Prix Gold Badminton Title 2018 in Men’s Singles Titles? 

(A) Srikanth Kidambi 

(B) Ajay Jayaram 

(C) Sameer Verma 

(D) Prannoy Kumar 

Answer: (C) 

 

Q. 47 Baking soda, the common cooking ingredient, has which of the following chemical name? 

(A) Sodium bicarbonate 

(B) Sodium chloride 

(C) Monosodium glutamate 

(D) Sodium benzoate 

Answer: (A) 

 

Q. 48 Union Cabinet has decide to raise contribution of Central Government to National Pension System (NPS)corpusofits employee from 10% to ……….. 

(A) 11% 

(B) 20% 

(C) 14% 

(D) 12% 

Answer: (C) 

 

Q. 49 ‘Bishan Singh Bedi’ is a legendary Indian player of: 

(A) cricket 

(B) hockey 

(C) football 

(D) lawn tennis 

Answer: (A) 

 

Q. 50 What is the SI unit for the measurement of pressure? 

(A) Newton 

(B) Joule 

(C) Ampere 

(D) Pascal 

Answer: (D) 

Quant 

Instructions 

For the following questions answer them individually 

Q. 51 A shopkeeper marks the price of an article 25 % above its cost price. After allowing 10% discount on the market price, the article is sold for ₹720. What is the difference (in ₹) between the cost price and market price of the article? 

(A) 160 

(B) 120 

(C) 180 

(D) 150 

Answer: (A) 

Explanation: 

Let the cost price be x 

marked price=1.25x 

Discount allowed=10% 

MP-discount=SP 

Therefore SP=0.9*1.25x 

Given SP=720 

(9/10)*(5/4)x=720 

x=80*8 

x=Rs 640 

MP-CP=1.25x-x=0.25x 

=640/4 

=Rs 160 

 

Q. 52 How many spherical balls each of 2cm radius can be made out of a solid metallic cube of edge 44 cm?(Take π = 22/7) 

(A) 2420 

(B) 2532 

(C) 2448 

(D) 2541 

Answer: (D) 

 

Q. 53 A sum of ₹6900 was lent partly at 5% and the rest at 8% simple interest. Total interest received after 3 years from both was ₹1359. What was the ratio of money lent at 5% and 8%? 

(A) 10 : 13 

(B) 11 : 12 

(C) 14 : 9 

(D) 31 : 38 

Answer: (B) 

 

Q. 54 The average ages of boys and girls in a class is 14.5 years and 15 years respectively. If the ratio of the number of boys and girls is 2 : 3, then what is the average age(in years) of all students in the class? 

(A) 15.2 

(B) 15.4 

(C) 14.8 

(D) 14.6 

Answer: (C) 

Explanation: 

let the number of boys be 2x and girls be 3x 

Sum of ages of all the boys=14.5*2x 

=29x 

Sum of ages of all the girls=15*3x 

=45x 

Sum of ages all the boys and girls=29x+45x 

=74x 

Total strength of the class=2x+3x=5x 

Average=74x/5x 

=14.8 

 

Q. 55 Note-books are bought at 16 for ₹700 and sold at 11 for ₹500. How many notebooks should be sold to gain ₹450? 

(A) 272 

(B) 260 

(C) 254 

(D) 264 

Answer: (D) 

Explanation: 

Cost price of each book =Rs 700/16 

Selling price of each book=Rs 500/11 

Profit on each book=(500/11)-(700/16) 

=100(3/176) 

Total profit=450 

let the number of books required be ‘n’ 

Therefore n*300/176 =450 

n=88*3 

n=264 books 

 

Q. 56 The compound interest on a sum of ₹15800 for 2 years at 9% per annum, when the interest is compound 8 monthly,is (nearest to a rupee): 

(A) ₹2992 

(B) ₹3016 

(C) ₹3018 

(D) ₹2956 

Answer: (C) 

 

Q. 57 Amina saves 16% of her income. Now her income is increased by 20 % but she still saves the same amount as before. What is the percentage increase in her expenditure? 

(A) 20.4 

(B) 24.2 

(C) 23.8 

(D) 21.6 

Answer: (C) 

Explanation: 

let the income be Rs 100 

Savings=Rs 16 

Expenditure=100-16 

=Rs 84 

New income=100*1.2 

=Rs 120 

Savings =Rs 16 

Expenditure=Rs 104 

Percentage increase in the expenditure=((104-84)/84)*100 

=2000/84 

=23.8% 

 

Q. 58 Walking at 75% of his usual speed, Raghu is 80 minutes late to his office. What is his usual time in hours to cover the same distance? 

(A) 3½  

(B)

(C)

(D) 4½ 

Answer: (C) 

 

Q. 59 A can do ⅗th of a work in 12 days, B can do ⅓rd of that work in 15 days. They worked together for 12 days and then A left the work , B alone will complete the remaining work  in? 

(A) 9 days 

(B) 6 days 

(C) 4 days 

(D) 5 days 

Answer: (B) 

Explanation: 

Given A does (3/5)th of work in 12 days 

so A takes 12*5/3 =20 days for complete work 

similarly B takes 15 days for (1/3)rd work and so for complete work it takes 45 days 

LCM of 20 and 45 is 180 units 

Each day ‘A’ does 180/20 =9 units of work 

Each day ‘B’ does 180/45 = 4 units of work 

So each day both can complete 13 units of work 

In 12 days they complete 12*13=156 units of work 

So 180-156-24 units 

B can complete 24 units in 24/4 =6 days 

 

Q. 60 Study the following table, (which shows the number of units of an item manufactured and sold by 5 companies) and answer the question. 

Which Company sold the highest percentage of units as compared to manufactured by it? 

(A) B

(B)

(C)

(D)

Answer: (C) 

Explanation: 

Percentage of units sold compared to manufactured by A=(30/42)*100=71.42 

Percentage of units sold compared to manufactured by B=(32/45)*100=71.11 

Percentage of units sold compared to manufactured by C=(26/36)*100=72.22 

Percentage of units sold compared to manufactured by E=(40/54)*100=74.07 

Answer is E 

 

Q. 61 The volume of a right circular cone is 2464 cm3 If the radius of its base is 14 cm, then its curved surface area (in cm2 ) is: 

(Take π = 22/7) 

(A) 88√58

(B) 44√58

(C) 44√85

(D) 88√85

Answer: (D) 

 

Q. 62 The diagonal of a rectangle field is 15 m and its area is 108 m2. What is the cost of fencing the field at ₹50.50 per m? 

(A) ₹2121 

(B) ₹2020 

(C) ₹1919 

(D) ₹2222 

Answer: (A) 

 

Q. 63 If x is subtracted from each other of 17, 14, 22 and 18 then these numbers are in proportion, in this order. What is the mean proportion between 9x and 5x? 

(A) 3√5 

(B)

(C)

(D) 6√5 

Answer: (D) 

 

Q. 64 The average of 15 numbers is 56. The average of first 8 numbers is 54.4 and that of last 8 numbers is 56.6. If 8th number is excluded then what is the average of the remaining numbers? (correct to one decimal place) 

(A) 56.6 

(B) 53.8 

(C) 53.6 

(D) 56.2 

Answer: (A) 

Explanation: 

Sum of 15 numbers=15*56=840 

Sum of first 8 numbers=54.4*8=435.2 

Sum of last 8 numbers=56.6*8=452.8 

Sum of both=435.2+452.8=888 

The eight number in the series=888-840 

=48 

Sum of the other 7 numbers=840-48 

=792 

Average of 14 numbers=792/14 

=56.6 

 

Q. 65 The value of: 

3/8  of 4/5 ÷ 6/5 + 3/7 of 7/12 ÷ 1/40 of 2/5 – 11/3 ÷11/30 of 2/3

(A) 10¼ 

(B)

(C) 2½ 

(D) 3⅓ 

Answer: (A) 

 

Q. 66 Two trains start at the same time, P from A to B and Q from B to A. If they arrive at B and A, respectively, 2½ hours and 10 hours after they passed each other, and the speed of P is 90 km/hr, then the speed of Q in kin/hr is? 

(A) 80 

(B) 75 

(C) 45 

(D) 60 

Answer: (C) 

 

Q. 67 What is the average of the mean, median and range of the data: 

34, 41, 6, 13, 27, 20, 48, and 55 ? 

(A) 30 ½ 

(B) 30 ⅓ 

(C) 36 ⅔ 

(D) 36 ⅓ 

Answer: (C) 

 

Q. 68 Working together A, B and C can complete a piece of work in 24 days. After working together for 4 days. C left the work. A and B completed the remaining work in 30 days. The number of days taken by C alone to complete the same work is: 

(A) 60 

(B) 54 

(C) 72 

(D) 90 

Answer: (C) 

Explanation: 

Let the total work be 48 units 

Number of units of work they together do on each day=48/24 =2 units 

In 4 days they together do 4*2=8 units of work 

So 48-8=40 units is left 

This is done by (A) and (B) in 30 days and so their combined efficiency=40/30 

=4/3 

In each day (C) can do 2-(4/3)=2/3 units 

So 48 units is done in 48*3/2=72 days 

 

Q. 69 A sum of ₹ x is divided among A, B and C such that the ratio of shares of A and B is 3 : 4 and that of B and C is 5: 6. If C receives ₹1440 more than what A receives, then the value of x is? 

(A) 9280 

(B) 9000 

(C) 9440 

(D) 9600 

Answer: (C) 

Explanation: 

A:B=3:4 

A:B=15:20 

B:C=5:6 

B:C=20:24 

A:B:C=15:20:24 

sum =15a+20a+24a=59a 

24a-15a=1440 

9a=1440 

a=160 

Total=59*160 

=9440 

 

Q. 70 Let x be the least number which when divided by 10, 12, 14 and 16 leaves the remainders 2, 4, 6 and 8, respectively, but x is divisible by 17. When x is divided by 52, the quotient is: 

(A) 96 

(B) 92 

(C) 95 

(D) 99 

Answer: (A) 

Explanation: 

The given number leaves the same negative remainder when divided by 2,4,6 and 8 i.e 2-10,4-12,6-14,8-16=-8 So the given number will be in the form of LCM(10,12,14,16)k-8 

=1680k-8 

for k=1,2 the number obtained is not divisible by 17 

for k=3 we have 1680*3-8 

=5040-8 

=5032 

Required number is 5032/52=96.76 

 

Q. 71 Two alloys A and B contain copper and zinc in the ratio 7 : 2 and 5 : 3 respectively. How many kg of A and B must be melted in order to get an alloy of 44 kg containing copper and Zinc in the ratio 3 : 1? 

(A) 24 kg, 20 kg 

(B) 30 kg, 14 kg 

(C) 28 kg, 16 kg 

(D) 36 kg, 8 kg 

Answer: (D) 

Explanation: 

Copper in the (A) alloy=7/9 

Copper in the (B) alloy=5/8 

Copper in the resultant mixture=3/4 

let the ratio in which they are mixed is x 

By using the principle of allegation we have 

x=((3/4)-(5/8))/((7/9)-(3/4)) 

x=(1/8)/(1/36) 

x=9:2 

Given 44 kg of mixture and so zinc copper present is (3/4)*44=33 kg 

Therefore (7/9)*9y +(5/8)*2y =33 

7y+(5y/4)=33 

33y/4 =33 

y=4 

9y=36 kgs and 2y=8 kgs 

 

Q. 72 Study the following table, (which shows the number of units of an item manufactured and sold by 5 companies) and answer the question. 

The ratio of the number of units manufactured by companies C and E together and the number of units sold by A and D together is? 

(A) 15 : 11 

(B) 17 : 15 

(C) 24 : 13 

(D) 15 : 13 

Answer: (A) 

Explanation: 

total number of units manufactured by C and E together is 36+54=90 units 

total number of units sold by A and D together is 30+36=66 units 

ratio=90/66 

=15:11 

 

Q. 73 A person bought 80 m cloth at ₹110 per m and sold 75 % of it at a gain of 6 %. At what gain percent did he sell the remaining cloth, if he gained 10% on his outlay? 

(A) 26% 

(B) 22% 

(C) 28% 

(D) 24% 

Answer: (B) 

Explanation: 

Given that 1m cost= Rs 110 

Total cost price=110*80 

=8800 

Total profit =10% 

Therefore total SP=1.1 CP 

=1.1 *8800 

=Rs 9680 

75% of cloth=80*(3/4) 

=60m 

It is sold at 6% profit so CP of 60m cloth=60*110=6600 

SP of 60m cloth=1.06*6600 

=Rs 6996 

SP of remaining cloth= 9680-6996 

=Rs 2684 

CP of 20m cloth=20*110=2200 

Profit percent=((2684-2200)/2200)*100 

=484/22 

=22% 

 

Q. 74 Study the following table, (which shows the number of units of an item manufactured and sold by 5 companies) and answer the question. 

What percent of total number of units sold by companies A, B and D is equal to the number of units manufactured by company C? (correct to one decimal place) 

(A) 33.3 

(B) 35.6 

(C) 40.9 

(D) 36.7 

Answer: (D) 

Explanation: 

Total units sold by A,(B) and D=30+32+26 

=98 units 

Total units manufactured by C=36 

Therefore 98=100% 

36=(36*100)/98 

=3600/98 

=36.7 

 

Q. 75 [9.5 ÷ (0.6 × 0.75 + 0.8 ÷ 16) + 0.75] ÷ (0.03 ÷ 0.6 of 0.01) 

The value of lies between: 

(A) 2 and 3 

(B) 1 and 2 

(C) 0 and 1 

(D) 3 and 4 

Answer: (D) 

Explanation: 

[9.5 ÷ (0.6 × 0.75 + 0.8 ÷ 16) + 0.75] ÷ (0.03 ÷ 0.6 of 0.01) 

= [9.5 ÷ (0.6 × 0.75 + 0.05) + 0.75] ÷ (5) 

= [9.5 ÷ (0.45 + 0.05) + 0.75] ÷ (5) 

= [19 + 0.75] ÷ (5) 

=(19.75/5) 

=3.95 

English 

Instructions 

For the following questions answer them individually 

Q. 76 Select the most appropriate word to fill in the blank. 

A successful entertainer ………….. the right mood before beginning a joke or story. 

(A) spreads 

(B) engages 

(C) creates 

(D) enjoys 

Answer: (C) 

Instructions 

In the following passage some words have been deleted. Fill in the blanks with the help of the alternatives given. 

Passage: 

After my job was over, I went to Mumbai and returned after (1)………… six months. I had barely taken (2)…………. my clothes when I saw Ratan standing (3)…………. a new baby. He was(4)…………. with joy. Even my daughter could not have (5)…………. such joy on finding her younger brother after several years. 

 

Q. 77 Select the most appropriate option that will fill in the blank number 1. 

(A) nearly 

(B) rarely 

(C) hardly 

(D) merely 

Answer: (A) 

 

Q. 78 Select the most appropriate option that will fill in the blank number 2. 

(A) away 

(B) over 

(C) of 

(D) off 

Answer: (D) 

 

Q. 79 Select the most appropriate option that will fill in the blank number 3. 

(A) with 

(B) by 

(C) near 

(D) along 

Answer: (A) 

 

Q. 80 Select the most appropriate option that will fill in the blank number 4. 

(A) bursting 

(B) singing 

(C) dancing 

(D) skipping 

Answer: (A) 

 

Q. 81 Select the most appropriate option that will fill in the blank number 5. 

(A) felt 

(B) advised 

(C) thought 

(D) understood 

Answer: (A) 

Instructions 

For the following questions answer them individually 

 

Q. 82 Select the most appropriate word to fill in the blank. 

The soldiers were instructed to ……….. restraint and handle the situation peacefully. 

(A) exercise 

(B) prevent 

(C) resist 

(D) control 

Answer: (A) 

 

Q. 83 Select the most appropriate option to substitute the underlined segment in the given sentence. If there is no need to substitute it, select No improvement. 

The GATT Treaty,it is explained, would allow subsidised to the extent of 10% of the market price. 

(A) would allow supply 

(B) would allow support 

(C) would allow subsidy 

(D) No improvement 

Answer: (C) 

 

Q. 84 From the given options, identify the segment in the sentence which contains the grammatical error. 

The speech he made on his visit to our college concerning our society’s needs have bothered me greatly. 

(A) The speech 

(B) have bothered 

(C) on his visit 

(D) concerning our 

Answer: (B) 

 

Q. 85 From the given options, identify the segment in the sentence which contains the grammatical error. 

He want to finish the assignment in the last week but in fact he could not. 

(A) he could not 

(B) in the last week 

(C) want to finish 

(D) the assignment 

Answer: (C) 

 

Q. 86 Select the correctly spelt word. 

(A) Equivalent 

(B) Equivelent 

(C) Equivalant 

(D) Eqivalent 

Answer: (A) 

 

Q. 87 From the given options, identify the segment in the sentence which contains the grammatical error. Every one of the survivor of the capsized ship has told the samestory. 

(A) the same story 

(B) Every one 

(C) capsized ship 

(D) of the survivor 

Answer: (D) 

 

Q. 88 Select the word which means the same as the group of words given. 

One who knows everything 

(A) Omniscient 

(B) Omnipotent 

(C) Onmnipatient 

(D) Onmnipresent 

Answer: (A) 

 

Q. 89 Select the correctly spelt word. 

(A) Throughfare 

(B) Thoroghfare 

(C) Thoroughfare 

(D) Thoroufare 

Answer: (C) 

 

Q. 90 Select the option that means the same as the given idiom. 

To sound a red alert. 

(A) To wave a red flag 

(B) To press a red button 

(C) To raise an alarm 

(D) To hear an alarm 

Answer: (C) 

 

Q. 91 Select the synonym of the given word. 

GALLANT 

(A) Wealthy 

(B) Violent 

(C) Courageous 

(D) Wild 

Answer: (C) 

 

Q. 92 Select the most appropriate option to substitute the underlined segment in the given sentence. If there is no need to substitute it, select No improvement. 

Dancing around a totem pole was one of the favourite pastimes of the American Indians. 

(A) was one of the favourite pastime 

(B) are the favourite pastimes 

(C) No improvement 

(D) were one of the favourite pastimes 

Answer: (C) 

 

Q.93 Select the most appropriate option to substitute the underlined segment in the given sentence. If there is no need to substitute it, select No improvement. 

Operation Flood was started with the primary objective for linking rural producers with urban consumers. 

(A) was started with the primary objectives of 

(B) No improvement 

(C) was started with primary objective 

(D) was started with the primary objective of 

Answer: (D) 

 

Q. 94 Select the option that means the same as the given idiom. 

In the good books 

(A) To be in favour of 

(B) To accompany good people 

(C) To collect good books 

(D) To do favourable things 

Answer: (A) 

 

Q. 95 Select the word which means the same as the group of words given. 

A city which is multicultural 

(A) Metro 

(B) Cosmopolitan 

(C) Urban 

(D) Global 

Answer: (B) 

 

Q. 96 Select the antonym of the given word. 

HOSTILE 

(A) Friendly 

(B) Optimistic 

(C) Negative 

(D) Serene 

Answer: (A) 

 

Q. 97 Select the most appropriate option to fill in the blank. 

Invite everyone in the group to ……….. us for a weekend party. 

(A) join 

(B) join together 

(C) join them 

(D) join each other 

Answer: (A) 

 

Q. 98 Select the most appropriate option to fill in the blank. 

If you listened carefully, you ……….. 

(A) would understand 

(B) will understand 

(C) would have understood 

(D) understand 

Answer: (A) 

 

Q. 99 Select the antonym of the given word. 

DUBIOUS 

(A) Docile 

(B) Certain 

(C) Doubtful 

(D) Faithful 

Answer: (B) 

 

Q. 100 Select the synonym of the given word. 

IMPAIR 

(A) Improve 

(B) Combine 

(C) Introduce 

(D) Weaken 

Answer: (D) 

SSC GD 19 Feb 2019 Shift-I Previous Year Paper

 SSC GD 19th Feb 2019 Shift-I 

Reasoning 

Instructions 

For the following questions answer them individually 

Q. 1 How would the pattern given on the square transparent sheet marked ‘X’ look when the sheet is folded along the dotted line? 

Answer: (A) 

Explanation: 

When we fold the sheet along the dotted line, the resulting figure will be the mirror image of the original figure, thus the triangle originally pointing to the left will now point to the right, hence last option is eliminated. 

As there is only a circle inside the triangle, thus in the answer figure there will only be a circle along the horizontal line (which is already present on the left side of X), thus only possible answer is the first one. 

=> Ans – (A) 

 

Q. 2 Four words have been given out of which three are alike in some manner, while one is different. Choose the odd one 

(A) Joy 

(B) Fear 

(C) Anger 

(D) Calm 

Answer: (D) 

Explanation: 

All the words except calm are emotions, hence calm is the odd one out. 

=> Ans – (D) 

 

Q. 3 The statements below are followed by two conclusions labeled I and II. Assuming that the information in the statements is true, even if it appears to be at variance with generally established facts, decide which conclusion(s) logically and definitely follow(s) from the information given in the statements. 

Statements: 

1. Some flowers are white. 

2. Some white objects are round. 

Conclusions: 

I. Some flowers are round. 

II. Every round object is either white or a flower. 

(A) Only conclusion II follows 

(B) Both conclusions I and II follow 

(C) Only conclusion I follows 

(D) Neither conclusion I nor II follows 

Answer: (D) 

Explanation: 

The venn diagram for above statements is : 

Conclusions: 

I. Some flowers are round = false 

II. Every round object is either white or a flower = false 

Thus, neither conclusion I nor II follows. 

=> Ans – (D) 

 

Q. 4 If in a code language, HEAD = 4158 and PASTE = 52019116, then TRICK = ? 

(A) 11391820 

(B) 31191820 

(C) 11391718 

(D) 11932018 

Answer: (A) 

 

Q. 5 DAWN is related to DUSK in the same way as INAUGURATION is related to: 

(A) MEETING 

(B) VALEDICTION 

(C) CONCLUSION 

(D) INVITATION 

Answer: (B) 

Explanation: 

Dawn is opposite of Dusk, similarly antonym of Inauguration is Valediction. 

=> Ans – (B) 

 

Q. 6 Six houses A, B, C, D, E and F are located in two rows facing each other with three houses in each row. F is opposite C, which is to the left of E. D is to the right of A. Which house is facing B? 

(A) (D) 

(B) (A) 

(C) (C) 

(D)

Answer: (B) 

Explanation: 

House C is to the left of E and F is opposite of C. 

This means that FDA are in same row and BCE are in another row. 

D is to the right of A, i.e. F is at one of the ends. Thus pattern is : 

=> House (A) is facing B. 

=> Ans – (B) 

 

Q. 7 Select the correct mirror image of the following figure when the mirror is placed to its right. 

Answer: (D) 

Explanation: 

When a vertical mirror is placed, object on the left will appear on the right inversed, and vice-versa. So, the trapezium on the right will appear on the left, but the longer un parallel side will remain on the top, thus first option is eliminated. 

Similarly, the square at top left will still have vertical lines inside it, but will now appear at top right, as shown in the last image. => 

Ans – (D) 

 

Q. 8 Select the correct option that will fill in the blank and complete the series. 

2, 3, 4, 8, 16, 25, 52, ……. 

(A) 114 

(B) 85 

(C) 68 

(D) 79 

Answer: (C) 

 

Q. 9 Choose the figure from the options that would follow next in the given series. 

Answer: (C) 

 

Q. 10 Select the option that is related to the third term in the same way as the second term is related to the first term. 

BDFH : AFCL :: FHJL : ? 

(A) EIGP 

(B) EJFO 

(C) HFGN 

(D) EJGP 

Answer: (D) 

 

Q. 11 Which of the following Venn diagrams correctly represents the relationships among the classes: 

Educated, Mothers, Employed 

Answer: (D) 

 

Q. 12 Four numbers have been given out of which three are alike in some manner, while one is different. Choose the odd one. 

(A) 84 

(B) 91 

(C) 42 

(D) 55 

Answer: (D) 

Explanation: 

Apart from 55, all other numbers are multiples of 7, hence it is the odd one. 

=> Ans – (D) 

 

Q. 13 If in a code language SYMPHONY is coded as ZOPIQNZT, then THURSDAY would be coded as: 

(A) UIVSTEBZ 

(B) ZBETSIVU 

(C) XZCRQTGS 

(D) ZBETSVIU 

Answer: (D) 

 

Q. 14 Select the correct option that will fill in the blank and complete the series. 

2, 5, 7, 12, ?, 31, 50 

(A) 18 

(B) 19 

(C) 25 

(D) 21 

Answer: (B) 

Explanation: 

The above series is an example of fibonacci series, where the next term is the sum of previous 2 terms. 

2 + 5 = 7 

5 + 7 = 12 

7 + 12 = 19 

12 + 19 = 31 

19 + 31 = 50 

=> Ans – (B) 

 

Q. 15 The statements below are followed by two conclusions labeled I and II. Assuming that the information in the statements is true, even if it appears to be at variance with generally established facts, decide which conclusion(s) logically and definitely follow(s) from the information given in the statements. 

Statements: 

1. Some mangoes are fruits. 

2. All fruits are sweet. 

Conclusions: 

I. Some sweet objects are mangoes. 

II. All sweet objects are fruits. 

III. Some mangoes are neither fruits nor sweet. 

(A) Only conclusions I and III follow 

(B) Only conclusions II and III follow 

(C) Only conclusions I and II follow 

(D) Only conclusion I follows 

Answer: (A) 

 

Q. 16 Select the option that is related to the third term in the same way as the second term is related to the first term. 

Torture : Cruelty :: Forgiveness : ? 

(A) Politeness 

(B) Generosity 

(C) Gratitude 

(D) Excuse 

Answer: (B) 

Explanation: 

Expression = Torture : Cruelty :: Forgiveness : ? 

Torture and cruelty have similar meanings, hence synonym of forgiveness is gratitude.

=> Ans – (B) 

 

Q. 17 Choose the figure from the options in which the figure marked ‘X’ is embedded. 

Answer: (C) 

 

Q. 18 Pick the odd pair out. 

(A) 14-41 

(B) 17-52 

(C) 23-32 

(D) 12-21 

Answer: (B) 

Explanation: 

In all the pairs except 17-52, the digits of the first number are reversed to form the second number, hence it is the odd one. 

=> Ans – (B) 

 

Q. 19 V, X, Y and Z are playing cards. X is to the left of Y and V is to the right of Z. If X is facing West, which direction is V facing? 

(A) North 

(B) South 

(C) West 

(D) East 

Answer: (A) 

 

Q. 20 Which two signs should be interchanged to make the following equation correct? 

12 + 16 ÷ 8 × 4 − 8 = 24 

(A) − and ÷ 

(B) × and − 

(C) ÷ and × 

(D) + and × 

Answer: (A) 

 

Q. 21 Which one of the following four-letter clusters does NOT belong to the group? 

(A) CXWD 

(B) FUSG 

(C) DWTG 

(D) BYWD

Answer: (B) 

Explanation: 

The pattern followed is that there are 2 pairs of symmetrically opposite alphabets in each term. Eg :- A and Z, B and Y, C and X, D and W and so on. 

But in the second option, (FU) and (SG) does not belong to that series, hence it is the odd one.

=> Ans – (B) 

 

Q. 22 Select the correct option that will fill in the blank and complete the series. 

cab, gef, ljk, rpq, ………. 

(A) ywx 

(B) yxw 

(C) zxy 

(D) wvu 

Answer: (A) 

Explanation: 

Expression : cab, gef, ljk, rpq, ………. 

The pattern followed in each letter of the terms is : 

1st letter : c (+4 letters) = g (+5 letters) = l (+6 letters) = r (+7 letters) =

2nd letter : a (+4 letters) = e (+5 letters) = j (+6 letters) = p (+7 letters) =

3rd letter : b (+4 letters) = f (+5 letters) = k (+6 letters) = q (+7 letters) =

Thus, missing term = ywx 

=> Ans – (A) 

 

Q. 23 Select the figure which when placed in the blank space of the figure marked ‘X’ would complete the pattern. 

Answer: (A) 

 

Q. 24 Among six objects P, Q, R, S, T and U, Q is heavier than R but lighter than T. S is lighter than T but heavier than U.P is lighter than Q but heavier than S. Which is the second heaviest object? 

(A)

(B)

(C)

(D)

Answer: (A) 

Explanation: 

Q is heavier than R but lighter than T, = T > Q > R 

S is lighter than T but heavier than U, = T > S > U 

P is lighter than Q but heavier than S, = Q > P > S 

Thus, final arrangement : T > Q > R, P > S > U 

Q is the second heaviest. 

=> Ans – (A) 

 

Q. 25 The numbers in the following set are related in a certain way. Choose the set that is similar to the following set: 

{6, 10, 8} 

(A) {8, 12, 9} 

(B) {5, 13, 12} 

(C) {13, 17, 16} 

(D) {3, 6, 4} 

Answer: (B) 

Explanation: 

The numbers in the set {6, 10, 8} are sides of right angled triangle, the middle number being the hypotenuse.

(6)2 + (8)2 = (10)2 

(5)2 + (12)2 = (13)2 

Similarly, only in the set {5, 13, 12}, => , the numbers represent sides of right angled triangle.

=> Ans – (B) 

General knowledge 

Instructions 

For the following questions answer them individually 

Q. 26 The famous playback singer ……….. is a recipient of the Bharat Ratna Award. 

(A) Shubha Mudgal 

(B) Asha Bhonsale 

(C) Shreya Ghoshal 

(D) Lata Mangeshkar 

Answer: (D) 

 

Q. 27 Which day is celebrated as the World Environment Day? 

(A) 5th June 

(B) 1st December 

(C) 5th August 

(D) 1st May 

Answer: (A) 

 

Q. 28 Who is an ‘indentured labourer’? 

(A) A slave from Africa 

(B) A labour bought in a market 

(C) An unpaid labourer 

(D) A bonded labourer, working to pay off his passage to a new country 

Answer: (D) 

 

Q. 29 ………….. is known as the Eastern Mountain Range. 

(A) Patkai Range 

(B) Aravalli 

(C) Karakoram 

(D) Shivalik 

Answer: (A) 

 

Q. 30 What do banks utilize a major portion of the deposits for? 

(A) Loans 

(B) Guarantee 

(C) Interest 

(D) Collaterals 

Answer: (A) 

 

Q. 31 In the context of Indian National Movement, who was known as ‘Frontier Gandhi’? 

(A) Maghfoor Ahmad Ajazi 

(B) Khan Abdul Gaffar Khan 

(C) Udham Singh 

(D) Ashfaquila Khan 

Answer: (B) 

 

Q. 32 What is diluted acetic acid commonly known as? 

(A) Oleum 

(B) Blue vitriol 

(C) Vinegar 

(D) Alum 

Answer: (C) 

 

Q. 33 The slogan “Workers of the world unite” was associated to: 

(A) French revolution 

(B) American revolution 

(C) Russian revolution 

(D) Japanese revolution 

Answer: (C) 

 

Q. 34 The ICC Emerging Player of the Year 2018 was awarded to ………… 

(A) Hardik Pandya 

(B) K.L. Rahul 

(C) Rishabh Pant 

(D) Shikhar Dhawan 

Answer: (C) 

 

Q. 35 The first amendment to the Indian Constitution was made in ………… 

(A) 1947 

(B) 1948 

(C) 1951 

(D) 1950 

Answer: (C) 

 

Q. 36 Which type of micro-organism causes typhoid? 

(A) Virus 

(B) Bacteria 

(C) Fungi 

(D) Protozoa 

Answer: (B) 

 

Q. 37 The Constitution of India contains ……….. schedules. 

(A) 11 

(B) 12 

(C) 14 

(D) 13 

Answer: (B) 

 

Q. 38 When a force resists the relative motion between two surfaces, it is called ……….. 

(A) Resistance 

(B) Convection 

(C) Friction 

(D) Induction 

Answer: (C) 

 

Q. 39 What is the movement of a plant due to the stimulus of light known as? 

(A) Respiration 

(B) Geotaxis 

(C) Geotropism 

(D) Phototropism 

Answer: (D) 

 

Q. 40 Malgudi Days was written by ……….. 

(A) Shashi Tharoor 

(B) Vikram Seth 

(C) Amish Tripathi 

(D) R.K. Narayan 

Answer: (D) 

 

Q. 41 Who won the National Squash 2018 Women’s Singles title? 

(A) Dipika Pallikal 

(B) Bhuvaneshkumari 

(C) Urwashi Joshi 

(D) Joshna Chinappa 

Answer: (D) 

 

Q. 42 River Yamuna does NOT pass through which of the following states? 

(A) Uttarakhand 

(B) Haryana 

(C) Gujarat 

(D) Uttar Pradesh 

Answer: (C) 

 

Q. 43 What is an annual statement of receipts and expenditure of the government over a fiscal year is known as? 

(A) Capital 

(B) Revenue 

(C) Tax 

(D) Budget 

Answer: (D) 

 

Q. 44 Which of the following pair correctly represents a state and the corresponding art form associated with it? 

(A) Assam — Kathakali 

(B) Tamil Nadu — Odissi 

(C) Uttar Pradesh — Kathak 

(D) Punjab — Mohiniyattam 

Answer: (C) 

 

Q. 45 Which one of the following is a labour intensive industry? 

(A) Pulses 

(B) Tea 

(C) Maize 

(D) Wheat 

Answer: (B) 

 

Q. 46 Under which of the following situations, the Fundamental Right to Freedom is curtailed in India? 

(A) Financial Emergency 

(B) Epidemic Emergency 

(C) State Emergency 

(D) National Emergency 

Answer: (D) 

 

Q. 47 Nalanda was an ancient center of learning for which religion? 

(A) Islam 

(B) Christianity 

(C) Jainism 

(D) Buddhism 

Answer: (D) 

 

Q. 48 Which of the following statement is NOT correct in the context of rising importance of tertiary sector in India? 

(A) Growth of information and technology 

(B) Growth in per capita income 

(C) Growth of service providing industries 

(D) Growth of political awareness in rural areas 

Answer: (D) 

 

Q. 49 Ustad Amjad Ali Khan is famous for playing the ………… 

(A) tabla 

(B) violin 

(C) veena 

(D) sarod 

Answer: (D) 

 

Q. 50 RK Laxman is famous for ……….. 

(A) wall painting 

(B) classical music 

(C) films 

(D) cartoons 

Answer: (D) 

Quant 

Instructions 

For the following questions answer them individually 

Q. 51 When a person goes to his office from his house with a speed of 10 km/hr, he is late by 20 minutes. When he goes with speed of 15 km/hr, he is late by 5 minutes. What is the distance (in km) between his office and house? 

(A)

(B) 7.5 

(C)

(D) 8.5 

Answer: (B) 

 

Q. 52 The value of: 

(A) 1/4

(B) 8

(C)

(D) 1/16 

Answer: (D) 

 

Q. 53 What is the sum of digits of the least number which when divided by 21, 28, 30 and 35 leaves the same remainder 10 in each case but is divisible by 17? 

(A) 11 

(B) 13 

(C) 14 

(D) 10 

Answer: (B) 

Explanation: 

L.C.M. (21,28,30,35) = 420 

Least number that will leave remainder 10 will be of the form = n = 1 

Now, if , number = 430, which is not divisible by 17 n = 2 

But, when , number = 850, which is divisible by 17 

420n + 10 

Thus, sum of digits of number 850 = => Ans – (B) 

8 + 5 + 0 = 13 

 

Q. 54 A park is in the shape of a rectangle. Its length and breadth 240 m and 100 m, respectively. At the center of the park, there is a circular lawn. The area of the park, excluding the lawn is 3904 m2. What is the perimeter (in m) of the lawn? (use π = 3.14) 

(A) 502.4 

(B) 516.2 

(C) 508.6 

(D) 512.8 

Answer: (A) 

 

Q. 55 The value of: 1.25 − [1 ÷ {3 + (2 − 0.4 × 2.5)}] 

(A)

(B)

(C)

(D) ½ 

Answer: (B) 

Explanation: 

1.25 − [1 ÷ {3 + (2 − 0.4 × 2.5)}] 

Expression = 

1.25 − [1 ÷ {3 + (2 − 1)}] 

= 1.25 − [1 ÷ (4)] 

= 1.25 − 0.25 = 1 

=> Ans – (B) 

 

Q. 56 The profit on selling 35 mangoes is equal to the cost price of 7 mangoes. What is the profit percentage? 

(A) 33⅓ 

(B) 20 

(C) 25 

(D) 16⅔ 

Answer: (B) 

 

Q. 57 A train takes 3 hours less time for a journey of 360 km, if its speed is increased by 10 km/hr from its usual speed. What is its increased speed (in km/ hr)? 

(A) 40 

(B) 42 

(C) 36 

(D) 30 

Answer: (A) 

 

Q. 58 After allowing a discount of 12½% on the marked price of an article. it was sold for ₹700. Had the discount NOT been given, the profit would have been 60%.The cost price of the article is: 

(A) ₹500 

(B) ₹540 

(C) ₹600 

(D) ₹480 

Answer: (A) 

 

Q. 59 If (x + 4), (x + 12), (x – 1), and (x + 5) are in proportion then the mean proportional between x and (x – 7)is: 

(A)

(B) 16 

(C) 15 

(D) 12 

Answer: (D) 

 

Q. 60 Three positive number are given. If the average of any two of them is added to the third number, the sums obtained are 172, 216 and 180. What is the average of the given three numbers? 

(A) 93 

(B) 95 ⅓ 

(C) 94 ⅔ 

(D) 96

Answer: (C) 

 

Q. 61 The selling price of an article is 84% of its cost price. If the cost price is increased by 20% and the selling price is increased by 25%, what is the percentage increase/decrease in the loss with respect to the earlier loss? 

(A) 6¼ %, decrease 

(B) 6⅔ %, decrease 

(C) 6¼%, increase 

(D) 6⅔ %, increase 

Answer: (A) 

 

Q. 62 2 men and 7 women can do a piece of work in 14 days whereas 3 men and 8 women can do it in 11 days. In how many days 5 men and 4 women can do the same work? 

(A) 11 

(B) 12 

(C) 14 

(D) 10 

Answer: (A) 

 

Q. 63 A metallic sphere of radius 4 cm is melted and the cast into small spherical balls, each of diameter 0.4 cm. The number of small balls will be: 

(A) 1000 

(B) 8000 

(C) 4000 

(D) 2000 

Answer: (B) 

 

Q. 64 A sum of ₹14460 is divided among A, B, C and D such that the ratio of share of A and B is 3 : 5, that of B and C is 6 : 7 and that of C and D is 14 : 15. What is the difference between the shares of A and C? 

(A) ₹1440 

(B) ₹1500 

(C) ₹2100 

(D) ₹2040 

Answer: (D) 

 

Q. 65 A pipe can fill a tank in 10 minutes while another pipe can empty it in 12 minutes. If the pipes are opened alternately each for 1 minute, beginning with the first pipe, the tank will be full after (in minutes): 

(A) 90 

(B) 109 

(C) 108 

(D) 120 

Answer: (B) 

 

Q. 66 Renu saves 30% of her income. If her savings increases by 30% and the expenditure increases by 25%, then the percentage increase in her income is: 

(A) 30 

(B) 25.8 

(C) 26.5 

(D) 15 

Answer: (C) 

 

Q. 67 Study the following table and answer questions: 

Number of employees working in five different companies. 

The ratio of the number of male employees in company C to the number of female employees in the company D is: 

(A) 3 : 2 

(B) 5 : 2 

(C) 4 : 3 

(D) 8 : 9 

Answer: (C) 

 

Q. 68 The compound interest on a sum of ₹15625 for years at 16% per annum, the interest compound annually is : 

(A) ₹6661 

(B) ₹5400 

(C) ₹7932 

(D) ₹7923 

Answer: (D) 

 

Q. 69 A and B are two cones. The curved surface area of A is twice that of B. The slant height of B is twice that of A. What is the ratio of radii of A to B? 

(A) 3:2 

(B) 1:4 

(C) 4:1 

(D) 2:1 

Answer: (C) 

 

Q. 70 Study the following table and answer questions: 

Number of employees working in five different companies. 

The number of female employees in company C is what percent more than the number of male employees in company E? 

(A) 55 

(B) 60 

(C) 48 

(D) 54 

Answer: (B) 

 

Q. 71 Two numbers are in the ratio 3 : 5. If 9 is subtracted from each, the numbers so obtained are in the ratio 12 : 23. If 3 is added to the smaller number and 7 is subtracted from the other number, then they will be in the ratio: 

(A) 5 : 6 

(B) 4 : 5 

(C) 3 : 4 

(D) 5 : 8 

Answer: (C) 

 

Q. 72 Study the following table and answer questions: 

Number of employees working in five different companies. 

What percent of total male employees in companies A and B is the number of female employees in E? 

(A) 35 

(B) 36 

(C) 30 

(D) 32 

Answer: (A) 

 

Q. 73 What is the mean of the range, median and mode of the data given below ? 

1, 2, 5, 9, 6, 3, 9, 7, 4, 3, 9, 1, 9, 6, 8, 1 

(A) 7½ 

(B)

(C) 8½ 

(D) 7

Answer: (A) 

 

Q. 74 The simple interest on a certain sum at 14% per annum for 3¼ years is ₹3731. What will the amount of the same sum for 5½ years at half the earlier rate?  

(A) ₹11931 

(B) ₹11913 

(C) ₹11537 

(D) ₹11357 

Answer: (D) 

 

Q. 75 The average height of a certain number of persons in a group is 155.5 cm. Later on 4 persons of height 154.6 cm, 158.4 cm, 152.2 cm and 153.8 cm leave the group. As a result the average height of the remaining persons increases by 0.15 cm. What was the number of persons initially in the group? 

(A) 18 

(B) 20 

(C) 22 

(D) 24 

Answer: (D) 

English 

Instructions 

For the following questions answer them individually 

Q. 76 From the given options, identify the segment in the sentence which contains the grammatical error. Had you told me that you were in Mumbai I could have certainly contacted you instead of waiting here. 

(A) you were 

(B) Had you 

(C) could have 

(D) instead of 

Answer: (C) 

 

Q. 77 Select the correctly spelt word. 

(A) Contemparary 

(B) Contemporary 

(C) Comtemporary 

(D) Contempory 

Answer: (B) 

 

Q. 78 Select the word which means the same as the group of words given. 

One who loves mankind 

(A) Humanist 

(B) Philologist 

(C) Philanthropist 

(D) Philosopher 

Answer: (C) 

 

Q. 79 Select the synonym of the given word. 

ABILITY 

(A) Disposition 

(B) Outlook 

(C) Instinct 

(D) Capacity 

Answer: (D) 

 

Q. 80 Select the antonym of the given word. 

VIRTUOUS 

(A) Worthy 

(B) Honest 

(C) Kind 

(D) Vicious 

Answer: (D) 

Instructions 

In the following passage some words have been deleted. Fill in the blanks with the help of the alternatives given. 

Passage: 

Fundamentally, all human beings possess a combination of fixed inherited traits. All individuals possess the ………….(1) highly developed nervous system, backbones, erect posture, hair,etc. Therefore, ………..(2) among human beings arise only in …………..(3) changes of this basic pattern. Racial differences …………..(4) one of the finest distinctions and are based only on ……….(5) slight differences. 

 

Q. 81 Select the most appropriate option that will fill in the blank number 1. 

(A) natural 

(B) large 

(C) same 

(D) different 

Answer: (C) 

 

Q. 82 Select the most appropriate option that will fill in the blank number 2. 

(A) similarities 

(B) naturalness 

(C) sameness 

(D) variations 

Answer: (D) 

 

Q. 83 Select the most appropriate option that will fill in the blank number 3. 

(A) minor 

(B) complete 

(C) large 

(D) fundamental 

Answer: (A) 

 

Q. 84 Select the most appropriate option that will fill in the blank number 4. 

(A) show 

(B) represent 

(C) give 

(D) present 

Answer: (B) 

 

Q. 85 Select the most appropriate option that will fill in the blank number 5. 

(A) hidden 

(B) possible 

(C) clear 

(D) certain 

Answer: (D) 

Instructions 

For the following questions answer them individually 

Q. 86 Fill in the blank with the most appropriate option. 

Devika is the most ……….. of them all and has managed to do well in her profession. 

(A) intelligent 

(B) simple 

(C) insolent 

(D) audacious 

Answer: (A) 

 

Q. 87 Select the option that means the same as the given idiom. 

Have several irons in the fire 

(A) Doing iron related work 

(B) Having too many engagements at a time 

(C) Burning unwanted pieces of iron 

(D) Talking to many persons simultaneously 

Answer: (B) 

 

Q. 88 Select the synonym of the given word. 

CHARACTER 

(A) Performance 

(B) Spirit 

(C) Mannerism 

(D) Trait 

Answer: (D) 

 

Q. 89 Select the antonym of the given word. 

TRANSPARENT 

(A) White-coloured 

(B) Translucent 

(C) Milky 

(D) Opaque 

Answer: (D) 

 

Q. 90 Select the most appropriate option to substitute the underlined segment in the given sentence. If there is no need to substitute it, select No improvement. 

The rules of chess require that one has to makes only one move at time. 

(A) made 

(B) has to make 

(C) will make 

(D) No improvement 

Answer: (B) 

 

Q. 91 Select the word which means the same as the group of words given. 

A person who is more than hundred years old 

(A) Centurial 

(B) Centuriator 

(C) Centurion 

(D) Centenarian 

Answer: (D) 

 

Q. 92 Select the correctly spelt word. 

(A) Incorigible 

(B) Incorrigible 

(C) Incourrigible 

(D) Incorrigibel 

Answer: (B) 

 

Q. 93 Select the most appropriate option to fill in the blank. 

He has been ………. my birthday all week! 

(A) celebrate 

(B) celebrating 

(C) celebrated 

(D) celebrates 

Answer: (B) 

 

Q. 94 Fill in the blank with the most appropriate option. 

Suresh is ………… in collecting stamps. 

(A) habitual 

(B) interested 

(C) customary 

(D) habituated 

Answer: (B) 

 

Q. 95 Select the most appropriate option to substitute the underlined segment in the given sentence. If there is no need to substitute it, select No improvement. 

He decided to leave very quickly the hotel. 

(A) leave the hotel quickly. 

(B) leave quickly the hotel. 

(C) quickly leave hotel. 

(D) No improvement 

Answer: (A) 

 

Q. 96 From the given options, identify the segment in the sentence which contains the grammatical error. If the report did not have the information need edit could not be used by them. 

(A) be used by them 

(B) it could not 

(C) If the 

(D) information needed 

Answer: (C) 

 

Q. 97 From the given options, identify the segment in the sentence which contains the grammatical error. The errors for the typed reports were so numerous that they could hardly be overlooked. 

(A) typed reports 

(B) could hardly be 

(C) so numerous 

(D) errors for 

Answer: (D) 

 

Q. 98 Select the most appropriate option to fill in the blank. 

………….. in Nagpur, we are unwilling to move to another city. 

(A) Since we have lived 

(B) Since we were living 

(C) Being that we are living 

(D) Since we have been living 

Answer: (D) 

 

Q. 99 Select the option that means the same as the given idiom. 

In high spirits 

(A) Become successful 

(B) Feel joyful 

(C) Create confusion 

(D) Remain calm 

Answer: (B) 

Q. 100 Select the most appropriate option to substitute the underlined segment in the given sentence. If there is no need to substitute it, select No improvement. 

Though a hero, he acted a coward. 

(A) like a coward 

(B) cowardly 

(C) No improvement 

(D) as coward 

Answer: (A) 

SSC GD 18 Feb 2019 Shift-III Previous Year Paper

SSC GD 18th Feb 2019 Shift-III 

Reasoning 

Instructions 

For the following questions answer them individually 

Q. 1 Select the option that is related to the third term in the same way as the second term is related to the first term. 

CGEI : XTVR :: FJHL : ? 

(A) UOQS 

(B) UQSO 

(C) TNRP 

(D) OUQS 

Answer: (B) 

 

Q. 2 How would the pattern given on the square transparent sheet marked ‘X° look when the sheet is folded along the dotted line? 

Answer: (C) 

 

Q. 3 Select the correct option that will fill in the blank and complete the series. 1, 3, 8, 19, 42, ……… 

(A) 89 

(B) 93 

(C) 85 

(D) 77 

Answer: (A) 

Explanation: 

The Correct Logic in above number series is. 

1, 3, 8,19, 42, ……… 

1*2+1 =3 

3*2+2=8 

8*2+3=9 

19*2+4=42 

42*2+5=89 

 

Q. 4 Which one of the following four-letter clusters does NOT belong to the group? 

(A) SOPQ 

(B) WTUV 

(C) HEFG 

(D) NKLM 

Answer: (A) 

Explanation: 

All three follow same pattern WTUV, HEFG, NKLM 

But SOPQ does not follow same pattern. 

Hence Correct Answer is Option (A) – SOPQ is odd one. 

 

Q. 5 If in a code language, MACE = 5 and MASTER = 7, then SOLUTION = ? 

(A)

(B) 11 

(C)

(D) 10 

Answer: (A) 

Explanation: 

MACE = 5 (No. of Alphabet in word 4 + 1) = 5 

MASTER = 7 (No. of Alphabet in word 6 + 1) =7 

Similarly, SOLUTION =9 (No. of Alphabet in word 8 + 1) =9 

Answer is 9. 

 

Q. 6 Pinky is taller than Priya but shorter than Reena.Riya is taller than Sheela, who is shorter than Priya. Reena is taller than Riya, who is taller than Pinky. Who is the shortest? 

(A) Priya 

(B) Sheela 

(C) Riya 

(D) Pinky 

Answer: (B) 

Explanation: 

1) Pinky is taller than Priya but shorter than Reena. 

Pinky > Riya 

2) Riya is taller than Sheela, 

Riya > Sheela 

3) (Sheela)who is shorter than Priya. 

Priya > Sheela 

4) Reena is taller than Riya, 

Reena> Riya 

5) (Riya)who is taller than Pinky. 

Riya > Pinky 

After consolidating all the sentence the Final Sequence of from Tallest to Shortest is Reena > Riya > Pinky > Priya > Sheela Hence Sheela is Shortest. 

 

Q. 7 Six friends A, B, C, D, E and F are sitting around a round table facing the center. C is seated between A and B, who is to the immediate left of D. If C is facing E, who is facing F? 

(A)

(B) A

(C)

(D)

Answer: (D) 

Explanation: 

1) B(who) is to the immediate left of D. 

2) C is seated between A and B 

3) C is facing E. 

4) Lets place F in Empty Space. 

Hence in (B) is facing F. 

 

Q. 8 Select the figure which when placed in the blank space of the figure marked ‘X’ would complete the pattern. 

Answer: (C) 

 

Q. 9 Which two signs should be interchanged to make the following equation correct? 

12 + 18 × 24 ÷ 6 − 3 = 19 

(A) × and ÷ 

(B) ÷ and + 

(C) + and × 

(D) − and ÷ 

Answer: (B) 

Explanation: 

Original Statement 

12 + 18 × 24 ÷ 6 − 3 = 19 

Let’s interchange symbols 

÷and+ 

12 ÷ 18 × 24 + 6 − 3 = 19 

4 ÷ 6 × 24 + 6 − 3 = 19 

4 × 4 + 6 − 3 = 19 

4 × 4 + 3 = 19 

16 + 3 = 19 

19 = 19 

Hence Option (B) is correct Answer. 

 

Q. 10 Select the correct option that will fill in the blank and complete the series. pmtz, rkvx, tixv, vgzt, ……… 

(A) xebr 

(B) zdcw 

(C) xeyv 

(D) yerb 

Answer: (A) 

Explanation: 

First digit of all number sequence is P+1,R+1,T+1,V+1 = X 

Second digit of all number sequence is M-1,K-1,I-1,G-1 = E 

Third digit of all number sequence is T+1,V+1,X+1,Z+1= (B) 

Fourth digit of all number sequence is Z-1,X-1,V-1,T-1 = R 

Hence last Sequence will have XEBR. 

 

Q. 11 The statement below is followed by three conclusions labeled I, II and III. You have to assume everything in the statement to be true, and then decide which of the three suggested conclusions logically follows for pursuing. 

Statements: 

1.) No school is an institution. 

2.) All establishments that are not institutions have buildings. 

Conclusions: 

I. Schools are not establishments. 

II. All schools have buildings. 

III. Schools do not have buildings. 

(A) Only conclusion III follows 

(B) Either conclusionI or II follow 

(C) Only conclusion I follows 

(D) Only conclusion II follows 

Answer: (D) 

 

Q. 12 P, Q, R, S, T and U are standing in a queue. Q is just ahead of P. S is just behind T. U is NOT near R, who is separated from Q by two persons. Who is standing at the front of the queue? 

(A)

(B)

(C)

(D)

Answer: (B) 

 

Q. 13 Choose the figure from the options that would follow next in the given series. 

Answer: (C) 

 

Q. 14 Four numbers have been given out of which three are alike in some manner, while one is different. Choose the odd one. 

(A) 33 

(B) 73 

(C) 53 

(D) 23 

Answer: (A) 

Explanation: 

73,53,23 are Prime Numbers. 

But 33 is not a prime number hence it is odd one. 

 

Q. 15 Select the option that is related to the third term in the same way as the second term is related to the first term. 

Heart : Veins :: River : ? 

(A) Water 

(B) Current 

(C) Ocean 

(D) Tributaries 

Answer: (D) 

Explanation: 

Heart contains Veins. 

Similarly, River contains Tributaries. 

 

Q. 16 Which of the following Venn diagrams correctly represents the relationships among the classes: 

Women, Mothers, Engineers 

Answer: (A) 

 

Q. 17 Pick the odd one out. 

(A) Rome 

(B) Paris 

(C) London 

(D) Spain 

Answer: (D) 

Explanation: 

Rome is capital of Italy, 

Paris is capital of France, 

London is capital of United Kingdom, 

But Spain is a country name of European it is not and capital. Hence Option (D) is correct Option. 

 

Q. 18 The statements below are followed by two conclusions labeled I and II. Assuming that the information in the statements is true, even if it appears to be at variance with generally established facts, decide which conclusion(s) logically and definitely follow(s) from the information given in the statements. 

Statements: 

1.) All cups are mugs. 

2.) All mugs are utensils. 

Conclusions: 

I. All cups are utensils. 

II. Some utensils are cups. 

(A) Only conclusion I follows 

(B) Neither conclusion I nor II follows 

(C) Both conclusions I and II follow 

(D) Only conclusion II follows 

Answer: (C) 

 

Q. 19 TIME is related to CLOCK in the same way as DISTANCE is related to: 

(A) KILOMETRE 

(B) ODOMETER 

(C) DESTINATION 

(D) SPACE 

Answer: (B) 

Explanation: 

Clock is use to display Time. 

Similarly, Odometer is use to measure Distance. 

Hence Option (B) is correct Option. 

 

Q. 20 If in a code language, MOTHER is coded as OMHTRE,then THURSDAY would be coded as: 

(A) HTRUDSYA 

(B) HTRUSDYA 

(C) HTRUSDAY 

(D) UHTDSRYA 

Answer: (A) 

Explanation: 

MOTHER is coded as OMHTRE (First alphabet is exchanged with second, Third is exchanged with fourth, fixth is exchanged with sixth) THURSDAY will be coded as ‘HTRUDSYA’ 

 

Q. 21 Pick the odd pair out. 

(A) 5-31 

(B) 3-23 

(C) 12-86 

(D) 8-58 

Answer: (A) 

 

Q. 22 Select the correct option that will fill in the blank and complete the series. 

6, 9, 18, 27, …., 81 

(A) 62 

(B) 36 

(C) 48 

(D) 54 

Answer: (D) 

 

Q. 23 The numbers in the following set are related in a certain way. Choose the set that is similar to the following set: {4, 6, 9} 

(A) {6, 8, 12} 

(B) {9, 15, 18} 

(C) {6, 12, 24} 

(D) {12, 14, 18} 

Answer: (C) 

 

Q. 24 Choose the figure from the options in which the given figure is embedded. 

Answer: (C) 

 

Q. 25 Choose the alternative which most closely resembles the mirror image of the given word when mirror is placed at XY line. 

Answer: (B) 

General knowledge 

Instructions 

For the following questions answer them individually 

Q. 26 Who was the first Indian woman player to reach the Olympics badminton singles quarterfinals? 

(A) PV Sindhu 

(B) Ashwini P 

(C) Aparna Popat 

(D) Saina Nehwal 

Answer: (D) 

 

Q. 27 Why is it important to have iodised salt in our diet? 

(A) It improves eye sight 

(B) It helps in smooth functioning of the thyroid gland 

(C) It helps in building bones 

(D) It helps in gaining height 

Answer: (B) 

 

Q. 28 As per the interim Budget 2019, the 22nd AIIMS is proposed to come up in ……… 

(A) Haryana 

(B) Assam 

(C) Maharashtra 

(D) Jharkhand 

Answer: (A) 

 

Q. 29 The Simon Commission which was sent by the British Government was boycotted because: 

(A) The Commission curbed financial independence of India. 

(B) The Commission did not have any Indian member. 

(C) There were differences amongst the members. 

(D) It gave special powers to the Princely states. 

Answer: (B) 

 

Q. 30 ‘Moonwalk’, the famous dance move,is associated with: 

(A) Michael Jackson 

(B) Adele 

(C) Misha Bryan 

(D) Fred Astaire 

Answer: (A) 

 

Q. 31 Who amongst the following founded the Bharatiya Jana Sangh (BJS) in 1977? 

(A) Ashutosh Mukherjee 

(B) Shyama Prasad Mukherjee 

(C) Vinayak Savarkar 

(D) Deen Dayal Upadhyaya 

Answer: (B) 

 

Q. 32 The Finance Minister of India, in his budget speech announced the launching of the indigenously developed semi high speed train named as ……… 

(A) Vikramsheela Express 

(B) Vande Bharat Express 

(C) Veer Jawan Express 

(D) Vaicom Express 

Answer: (B) 

 

Q. 33 The ………. movement was led by Muhammad Ali and Shaukat Ali. 

(A) Khilafat Movement 

(B) Badshahi Movement 

(C) Deoband Movement 

(D) Sultaniya Movement 

Answer: (A) 

 

Q. 34 Which of the following is not a Greenhouse gas? 

(A) Nitrous Oxide 

(B) Methane 

(C) Carbon Dioxide 

(D) Argon 

Answer: (D) 

 

Q. 35 Who called dams as the ‘Temples of Modern India’? 

(A) Mohandas Karamchand Gandhi 

(B) Sardar Vallabhbhai Patel 

(C) T Krishnanchari 

(D) Pt. Jawaharlal Nehru 

Answer: (D) 

 

Q. 36 Name the place where the Olympic Games originated? 

(A) Latvia 

(B) Baghdad 

(C) Greece 

(D) Aden 

Answer: (C) 

 

Q. 37 Fundamental duties were incorporated in the Indian Constitution by: 

(A) 56th Amendment 

(B) 42nd Amendment 

(C) 25th Amendment 

(D) 14th Amendment 

Answer: (B) 

 

Q. 38 The Right to Property was deleted from the list of Fundamental Rights by which of the Amendment of Indian Constitution? 

(A) 43rd Amendment 

(B) 41st Amendment 

(C) 42nd Amendment 

(D) 44th Amendment 

Answer: (D) 

 

Q. 39 Acidity in stomach after overeating can be remedied by consuming: 

(A) vegetables 

(B) fruit juice 

(C) lemon juice 

(D) baking soda 

Answer: (D) 

 

Q. 40 Where is the Konark Sun Temple situated? 

(A) Kedarnath 

(B) Odisha 

(C) Madurai 

(D) Tirupati 

Answer: (B) 

 

Q. 41 Sanjhi, the paper craft is said to have originated in which of the following places of India? 

(A) Bhubaneshwar 

(B) Coimbatore 

(C) Mathura 

(D) Indore 

Answer: (C) 

 

Q. 42 Name the acid present in tamarind. 

(A) Tartaric acid 

(B) Lactic acid 

(C) Amino acid 

(D) Citric acid 

Answer: (A) 

 

Q. 43 The objective of the 12th 5 year Plan was ………… 

(A) Faster, sustainable and more inclusive growth 

(B) Reducting poverty and exploitation of women 

(C) Growth with stability and progressive achievement 

(D) Faster and more inclusive growth 

Answer: (A) 

 

Q. 44 Which is the first Indian state to start rooftop rainwater harvesting? 

(A) Assam 

(B) Kerala 

(C) Tamil Nadu 

(D) Rajasthan 

Answer: (C) 

 

Q. 45 The Lahore session of the Indian National Congress declared to celebrate 26th January as the Independence day in the year ……… 

(A) 1942 

(B) 1935 

(C) 1930 

(D) 1920 

Answer: (C) 

 

Q. 46 Milk becomes sour if kept outside the refrigerator in summers because of: 

(A) its colour 

(B) the conversion of lactose sugar to lactic acid 

(C) the breakdown of proteins 

(D) the calcium present in it 

Answer: (B) 

 

Q. 47 Which of the following soils is found in the northern plains of India? 

(A) Arid soil 

(B) Alluvial soil 

(C) Laterite soil 

(D) Black soil 

Answer: (B) 

 

Q. 48 Robben Island, the world heritage site is associated to which of the following figures? 

(A) Nelson Mandela 

(B) Winston Churchill 

(C) Jimmy Carter 

(D) John F. Kennedy 

Answer: (A) 

 

Q. 49 Who replaced Sachchidananda Sinha as Chairman of the Constituent Assembly of India? 

(A) Pt. Jawaharlal Nehru 

(B) Dr. Rajendra Prasad 

(C) Sardar Vallabhbhai Patel 

(D) Mohandas Karamchand Gandhi 

Answer: (B) 

 

Q. 50 Who played a decisive role is the integration of the princely states of India? 

(A) Motilal Nehru 

(B) Sardar Vallabhbhai Patel 

(C) APJ Abdul Kalam 

(D) TT Krishnamurthy 

Answer: (B) 

Quant 

Instructions 

For the following questions answer them individually 

Q. 51 The compound interest on a certain sum at a certain rate percent per annum for the second year and the third year are ₹3300 and ₹3630, respectively. The sum is: 

(A) ₹32000 

(B) ₹28400 

(C) ₹30000 

(D) ₹25000 

Answer: (C) 

 

Q. 52 A boat can cover 36 km downstream and 16 km upstream in 10 hours. It can cover 3 km downstream and 1.2 km upstream in 48 minutes. What is the speed (in km/hr) of the boat when going upstream? 

(A)

(B) 2.5 

(C) 3.5 

(D)

Answer: (D) 

Explanation: 

let the speed of the boat=b 

speed of the stream=s 

Given (36)/(b+s) +(16)/(b-s) =10 

(36b-36s+16b+16s)/((b-s)(b+s))=10 

52b-20s=10(b-s)(b+s) 

Also given (3)/(b+s) +(1.2)/(b-s) =4/5 

(3b-3s+1.2b+1.2s)/((b-s)(b+s))=4/5 

4.2b-1.8s=(4/5)(b+s)(b-s) 

dividing both the equations we have 

104b-40s=105b-45s 

-b=-5s 

b=5s 

Substituting in the above equation 

(3)/(5s+s) +(1.2)/(5s-s) =4/5 

(1/2s)+(3/10s)=4/5 

8/10s =4/5 

s=1 km/hr 

b=5 km/hr 

Upstream speed=5-1 

=4 km/hr 

 

Q. 53 The value of [0.08 ÷ 1.2 of (3.4 − 2.6) × 0.8 of 3.2] of 9/16 lies between: 

(A) 0.11 and 0.13 

(B) 0.13 to 0.15 

(C) 0.09 and 0.11 

(D) 0.07 and 0.09 

Answer: (A) 

 

Q. 54 Study the following table and answer questions: 

Number of students studying in five schools of a city 

In how many schools, the number of girls is more than the average number of boys per school? 

(A)

(B)

(C)

(D)

Answer: (D) 

Explanation: 

Total number of boys in all the schools is 

(9/17)*680=360 

(4/9)*720=320 

(5/18)*540=150 

(2/3)*660=440 

(13/25)*900=468 

Sum=1838 

average=1838/5 =367 

Number of girls in each school=680-360=320 

720-320=400 

540-150=390 

660-440=220 

900-468=432 

Therefore three schools have the girls more than average boys per school. 

 

Q. 55 Two solutions of acid and water containing acid and water in the ratio 2 : 7 and 4 : 5, respectively are mixed in the ratio 2 : 5. What is the ratio of acid and water in the resulting solution? 

(A) 8 : 13 

(B) 3 : 7 

(C) 8 : 17 

(D) 2 : 1 

Answer: (A) 

Explanation: 

acid content in 1st solution 2/9 

acid content in second solution=4/9 

They are mixed in the ratio 2:5 

x is the acid present in the final mixture 

By using the principle of allegations we have 

((4/9)-x)/(x-(2/9))=2/5 

(20/9)-5x=2x-(4/9) 

7x=24/9 

x=8/21 

8 parts of acid and 13 parts of water. 

Ratio=8/13 

 

Q. 56 What is the mean of the range, median and mode of the data: 

2, 3, 1, 1, 3, 4, 2, 1, 1, 4, 5, 2, 4, 2, 2, 1, 3, 3, 2, 5 

(A) 3 ⅓ 

(B) 4 ½ 

(C) 2 ⅔ 

(D) 4

Answer: (C) 

 

Q. 57 Suman sold an article for 882 after allowing 16% discount on its marked price. Had she not allowed any discount, she would have gained 20% on the cost price. What is the cost price ofthe article? 

(A) 875 

(B) 850 

(C) 890 

(D) 880 

Answer: (A) 

Explanation: 

MP-discount=SP 

MP-0.16MP=882 

0.84MP=882 

MP=882*100/84 

Given in the second case MP=SP 

Therefore 1.2CP=SP 

CP=SP/1.2 

CP=(882*100)/(84*1.2) 

CP=875 

 

Q. 58 Two pipes A and B can fill a tank in 12 minutes and 15 minutes, respectively. When an outlet pipe C is also opened, then the three pipes together can fill the tank in 10 minutes. In how many minutes can C alone empty the full tank? 

(A) 18 

(B) 20 

(C) 16 

(D) 24 

Answer: (B) 

Explanation: 

Time taken for Pipe A to fill the tank=12 minutes 

Time taken for Pipe B to fill the tank=15 minutes 

Time taken for Pipe C to empty the tank=x minutes 

Therefore (1/12)+(1/15)-(1/x)=(1/10) 

(1/x)=(1/12)+(1/15)-(1/10) 

(1/x)=(5+4-6)/60 

(1/x)=3/60 

x=20 minutes 

 

Q. 59 A person covered a distance of 250 km at a certain speed. If his speed is 20% less, he would have covered the same distance in 1¼ hours more time. His speed (in km/hr), initially was: 

(A) 45 

(B) 50 

(C) 52½ 

(D) 47½ 

Answer: (B) 

 

Q. 60 When 32% of a number X was added to another number y , then y increases by 24%. The value of (2x+y)/3x−y is: 

(A)

(B)

(C)

(D)

Answer: (A) 

 

Q. 61 The average weight of a certain number of persons in a group was 75.5 kg. Later on, 4 persons weighing (2x+y)/3x−y joined the group. As a result, the average weight of all persons in the group reduced by 500g. The number of persons in the group, initially, was: 

(A) 20 

(B) 16 

(C) 18 

(D) 24 

Answer: (A) 

Explanation: 

Let the number of people be ‘x’ 

Sum of the weights=75.5x 

72.6 kg. 74 kg, 73.4 kg and 70 kg were added and average decreased to 75 

Sum=290 kgs 

Total sum=75.5x+290 

Average=(75.5x+290)/(x+4) 

(75.5x+290)/(x+4)=75 

75.5x+290=75x+300 

0.5x=10 

x=20 people 

 

Q. 62 Two numbers are in the ratio 5 : 6. If 16 is subtracted from each, the numbers will be in the ratio 3 : 4. If 8 is added to the first number and 3 is subtracted from the second, then they will be in the ratio: 

(A) 14 : 13 

(B) 16 : 15 

(C) 15 : 16 

(D) 13 : 14 

Answer: (B) 

Explanation: 

let the two numbers be 5x and 6x 

Given (5x-16)/(6x-16)=3/4 

20x-64=18x-48 

2x=16 

x=18 

Numbers are 40 and 48 

8 is added it becomes 48 and 3 is subtracted it becomes 45 

Ratio 48:45 

=16:15 

 

Q. 63 If the radius of a sphere is decreased by 25%, then the percentage decrease in its surface area is: 

(A) 42 

(B) 43.75 

(C) 37.50 

(D) 40 

Answer: (B) 

 

Q. 64 The area of an equilateral triangular park is equal to 5√3 times the area of a triangular field with sides 18 m, 80 m and 82 m. What is the side of the triangular park? 

(A) 125 m 

(B) 120 m 

(C) 140 m 

(D) 100 m 

Answer: (B) 

 

Q. 65 The value of is: 

 

(A) 1 ⅖ 

(B) 1 ⅘ 

(C) 3 ⅖ 

(D) 3 ⅕ 

Answer: (B) 

 

Q. 66 A sold an article to B at 25% profit. B sold it to C at a profit of 10% and C sold it to D at 40% loss. If I bought it for ₹46.20, then the difference between the profits of A and B is: 

(A) ₹8.50 

(B) ₹15 

(C) ₹6 

(D) ₹7 

Answer: (D) 

Explanation: 

Let the A’s CP=x 

A’s SP=1.25x 

B’s CP=1.25x 

B’s SP=1.1*1.25x 

C’s CP=1.1*1.25x 

C’s SP=0.6*1.1*1.25x 

D’s CP=0.6*1.1*1.25x=46.20 

0.825x=46.20 

x=46.20/0.825 

x=Rs 56 

A’s profit=0.25x=56/4=14 

B’s profit=0.125x=7 

Difference=14-7=Rs 7 

 

Q. 67 The curved surface area of a right circular cylinder of height 28 cm is 176 cm2 . The volume(in cm3 ) of cylinder is (Take π = 22/7) 

(A) 66 

(B) 110 

(C) 88 

(D) 176 

Answer: (C) 

 

Q. 68 Study the following table and answer questions: 

Number of students studying in five schools of a city 

The ratio of the number of boys in school (A) to the number of girls in school E is: 

(A) 3 : 4 

(B) 5 : 6 

(C) 8 : 13 

(D) 5 : 12 

Answer: (B) 

Explanation: 

the number of boys in school (A) =(9/17)*680 

=360 

the number of girls in school E =(12/25)*900 

=36*12 

=432 

Ratio=360:432 

=5:6 

 

Q. 69 Working together, A, B and C can complete a work in 36 days. If the efficiency of A, B and are C in the ratio 5: 6: 9, then B alone will complete the same work in: 

(A) 120 days 

(B) 100 days 

(C) 60 days 

(D) 90 days 

Answer: (A) 

Explanation: 

Given the ratios of efficiencies 5:6:9 and so 

A can do the work in (x/5) days 

B can do the work in (x/6) days 

C can do the work in (x/9) days 

Therefore 1/(x/5) + 1/(x/6)+ 1/(x/9) =1/36 

(5/x)+(6/x)+(9/x)=(1/36) 

20/x =1/36 

x=36*20 

x=720 

(B) can alone do in 720/6 =120 days 

 

Q. 70 A person sells a watch at a profit of 26%. If he had bought it at 20% less and sold for ₹81.60 less, he would have gained 32%. What is the original cost price (in ₹) of the watch? 

(A) 400 

(B) 480 

(C) 450 

(D) 360 

Answer: (A) 

Explanation: 

Let the cost price =x 

then selling price=1.26x 

Given that if he purchased for 20% less then cost price will be 0.8x 

Selling price is 81.6 less and so new selling price=1.26x-81.6 

given profit percent=32 

Therefore 1.32(0.8x)=1.26x-81.6 

1.056x=1.26x-81.6 

0.204x=81.6 

x=81.6/0.204 

x=400 

 

Q. 71 Let x be the greatest number such that when 12085, 16914 and 13841 are divided by it, the remainder in each case is same. The sum of digits of x is: 

(A) 14 

(B) 19 

(C) 13 

(D) 16 

Answer: (D) 

Explanation: 

As in this case each number leaves the same remainder we have to find the HCF of differences of the three numbers i.e 13841-12085=1756 

16914-13841=3073 

16914-12085=4829 

HCF of these three numbers is 439 

sum=4+3+9=16 

 

Q. 72 The simple interest on a certain sum in 5½ years at 6% p.a. is ₹ 2541. what will be the amount of the same sum in 5⅔ years at 817% p.a.? 

(A) ₹13125 

(B) ₹11235 

(C) ₹12153 

(D) ₹11253 

Answer: (D) 

Explanation: 

I=PTR/100 

2541=P*(11/2)*(6)/100 

P=(2541*200)/(66) 

P=Rs 7700 

I=PTR/100 

I=7700*(17/3)*(57/7)/100 

I=(7700*17*57)/(3*7*100) 

I=RS 3553 

A=P+I 

A=7700+3553 

A=11253 

 

Q. 73 Study the following table and answer questions: 

Number of students studying in five schools of a city 

The total number of girls in schools B and C is what percent less than the total number of boys in schools A and D? 

(A) 1.25 

(B) 11.25 

(C)

(D) 10 

Answer: (A) 

Explanation: 

The total number of girls in schools B and C=720*(5/9)+540*(13/18) 

=400+390 

=790 

the total number of boys in schools A and D=680*(9/17)+660*(2/3) 

=360+440 

=800 

percentage required=(10/800)*100 

=1.25% 

 

Q. 74 A sum of ₹4107 is divided between A, B, C and D such that the ratio of shares of A and B is 3 :4, that of B and C is 2: 1 and that of C and D is 3 : 5. The share of B is: 

(A) ₹1443 

(B) ₹1003 

(C) ₹1223 

(D) ₹1332 

Answer: (D) 

Explanation: 

Given a:b=3:4 

b:c=2:1 

b:c=4:2 

a:b:c=3:4:2 

c:d=3:5 

a:b:c=9:12:6 

c:d=6:10 

a:b:c:d=9:12:6:10 

Sum=9x+12x+6x+10x 

=37x 

Therefore 37x=4107 

x=4107/37 

x=111 

Share of (B) is 111*12=1332 

 

Q. 75 The average of 41 numbers is 63. The average of first 21 number is 61.5 and that of last 21 numbers is 65.5. if 21st number is excluded then the average of remaining numbers is: 

(A) 62.475 

(B) 63.5 

(C) 62.5 

(D) 63.575 

Answer: (A) 

Explanation: 

Sum of 41 numbers=63*41 

=2583 

Sum of first 21 numbers=21*61.5 

=1291.5 

Sum of last 21 numbers=21*65.5 

=1375.5 

Sum of 41 numbers and the 21st number=1291.5+1375.5 

=2667 

21st number in the series is =2667-2583 

=84 

Sum of 40 numbers=2583-84 

=2499 

Average=2499/40 

=62.475 

English 

Instructions 

For the following questions answer them individually 

Q. 76 Select the correctly spelt word. 

(A) Exaggeration 

(B) Exeggeration 

(C) Exaggaration 

(D) Exageration 

Answer: (A) 

 

Q. 77 Select the most appropriate option to fill in the blank. 

Scientists tried to find out the ……….. behind the occurrence of the phenomenon. 

(A) interpretation 

(B) cause 

(C) clarification 

(D) illustration 

Answer: (B) 

 

Q. 78 Select the correctly spelt word. 

(A) Accomodate 

(B) Accommodate 

(C) Acommodate 

(D) Accommadate 

Answer: (B) 

 

Q. 79 Select the most appropriate option to substitute the underlined segment in the given sentence. If there is no need to substitute it, select No improvement. 

The Indian forces are known for their bravery. 

(A) No improvement 

(B) force are known for 

(C) forces are known of 

(D) forces is known for 

Answer: (A) 

 

Q. 80 From the given options,identify the segment in the sentence which contains the grammatical error. Nowadays spectacles are costly than they used to be. 

(A) spectacles are 

(B) costly than 

(C) they used to be 

(D) Nowadays 

Answer: (B) 

 

Q. 81 From the given options, identify the segment in the sentence which contains the grammatical error. We must visit this ancient temple which are a hundred years old. 

(A) We must visit 

(B) this ancient temple 

(C) a hundred years old 

(D) which are 

Answer: (D) 

 

Q. 82 Select the most appropriate word to fill in the blank. 

Dinosaurs are an ………. species now. 

(A) age-old 

(B) extinct 

(C) antique 

(D) ancient 

Answer: (B) 

 

Q. 83 Select the most appropriate option to substitute the underlined segment in the given sentence.If there is no need to substitute it, select No improvement. 

Regular exercise lead to fitness of the body. 

(A) No improvement 

(B) leads to fitness of the body 

(C) leading to fitness of the body 

(D) will leads to fitness of the body 

Answer: (B) 

 

Q. 84 Select the most appropriate word to fill in the blank. 

Apart from cows, buffaloes are also ………. for mass milk production for humans. 

(A) reared 

(B) grown 

(C) produced 

(D) developed 

Answer: (A) 

Instructions 

In the following passage some words have been deleted. Fill in the blanks with the help of the alternatives given. 

Passage: 

The neglect of one half of mankind costs the world a great deal both, in economic and social(1)…….. (A) family (2)………….. an educated motheris, in most cases, a poorer family than one in which everyone is educated and can (3)…………. a living. (A) family with an uneducated mother may(4)………….. in other ways too, because the mother is (5)…………. to help the children in their education. 

Q. 85 Select the most appropriate option that will fill in the blank number 1. 

(A) terms 

(B) aspects 

(C) areas 

(D) values 

Answer: (A) 

 

Q. 86 Select the most appropriate option that will fill in the blank number 2. 

(A) with 

(B) of 

(C) without 

(D) including 

Answer: (C) 

 

Q. 87 Select the most appropriate option that will fill in the blank number 3. 

(A) Manage 

(B) eam 

(C) create 

(D) plan 

Answer: (B) 

 

Q. 88 Select the most appropriate option that will fill in the blank number 4. 

(A) suffer 

(B) destroy 

(C) fail 

(D) prosper 

Answer: (A) 

 

Q. 89 Select the most appropriate option that will fill in the blank number 5. 

(A) qualified 

(B) eligible 

(C) able 

(D) unable 

Answer: (D) 

Instructions 

For the following questions answer them individually 

 

Q. 90 Select the most appropriate option to fill in the blank. 

The baby bird was ……….. by its mother. 

(A) sacrificed 

(B) abandoned 

(C) thrown 

(D) renounced 

Answer: (B) 

 

Q. 91 Select the option that means the same as the given idiom. 

(A) stone’s throw away 

(A) At a short distance 

(B) Many miles away 

(C) At a remote location 

(D) Very far off 

Answer: (A) 

 

Q. 92 Select the antonym of the given word. 

LATENT 

(A) Forbidding 

(B) Obvious 

(C) Hidden 

(D) Artificial 

Answer: (B) 

 

Q. 93 From the given options, identify the segment in the sentence which contains the grammatical error. Dr. Rana is our teacher of English since this semester. 

(A) of English since 

(B) this semester. 

(C) Dr.Rana is 

(D) our teacher 

Answer: (A) 

 

Q. 94 Select the synonym of the given word. 

HANDY 

(A) Funny 

(B) Convenient 

(C) Nice 

(D) Easy 

Answer: (B) 

 

Q. 95 Select the synonym of the given word. 

NOMADIC 

(A) Wild 

(B) Barbaric 

(C) Brave 

(D) Roving 

Answer: (D) 

 

Q. 96 Select the most appropriate option to substitute the underlined segment in the given sentence.If there is no need to substitute it, select No improvement. 

We should offer due respects to all elderly persons. 

(A) offers due respects to 

(B) No improvement 

(C) offer due respects for 

(D) offer due respect to 

Answer: (D) 

 

Q. 97 Select the option that means the same as the given idiom. 

The grass is greener on the other side 

(A) Only green grass grows well 

(B) Green grass grows everywhere 

(C) Many things looks like green grass 

(D) Things appear better for others 

Answer: (D) 

 

Q. 98 Select the antonym of the given word. 

MALEVOLENT 

(A) Well-mannered 

(B) Positive 

(C) Kind 

(D) Cruel 

Answer: (C) 

 

Q. 99 Select the word which means the same as the group of words given. 

One who speaks many languages 

(A) Multilingual 

(B) Linguist 

(C) Bilingual 

(D) Monolingual 

Answer: (A) 

 

Q. 100 Select the word which means the same as the group of words given. 

Something which gets broken easily 

(A) Fragile 

(B) Feeble 

(C) Soft 

(D) Weak 

Answer: (A) 

SSC GD 18 Feb 2019 Shift-II Previous Year Paper

SSC GD 18th Feb 2019 Shift-II

Reasoning 

Instructions 

For the following questions answer them individually 

Q. 1 Select a figure from amongst the four alternatives that when placed in the blank space (?) of figure X will complete the pattern. (Rotation is not allowed). 

Answer: (B) 

Explanation: 

If we complete the above figure, then the missing pattern is represented by the red colour. 

=> Ans – (B) 

 

Q. 2 Choose the option in which the figure marked ‘X’ is embedded. (Rotation is not allowed) 

Answer: (B) 

Explanation: 

The question figure is embedded in following figure and is represented by red colour. 

=> Ans – (B) 

 

Q. 3 Choose the odd one out of the given options. 

(A) KJL 

(B) FIL 

(C) GIK 

(D) DKL 

Answer: (A) 

Explanation: 

Among the given options, only in KJL, there are consecutive letters from English alphabetical order. 

=> Ans – (A) 

 

Q. 4 A square transparent sheet with a pattern is given. How will the pattern appear when the transparent sheet is folded along the dotted line? 

Answer: (B) 

 

Q. 5 Select the option that is related to the third number in the same way as the second number is related to the first number. 

36 : 324 :: 11 : ? 

(A) 259 

(B) 121 

(C) 22 

(D) 221 

Answer: (C) 

Explanation: 

Expression = 36 : 324 :: 11 : ? 

The pattern followed is that the number on the left is multiplied by the sum of its digits. 

Eg :- 

36 × (3 + 6) = 324 

Similarly, 

11 × (1 + 1) = 11 × 2 = 22 

=> Ans – (C) 

 

Q. 6 The statements below are followed by two conclusions labeled I and II. Assuming that the information in the statements is true, even if it appears at variance with generally established facts, decide which conclusion(s) logically and definitely follow(s) from the information given in the statements. 

Statements: 

1) All toads are frogs. 

2) All frogs are snakes. 

Conclusion: 

I. Some snakes are toads. 

II. Some frogs are toads. 

(A) Only conclusion I follows. 

(B) Either conclusion I or conclusion II follows. 

(C) Both conclusions follow. 

(D) Only conclusion II follows. 

Answer: (C) 

 

Q. 7 Eight friends A, B, C, D, E, F, G and H are sitting around circular table facing each other for a lunch. A is opposite D and third to the right of B. G is between A and F. H is to the right of A. E is between C and D. Who is sitting opposite B? 

(A)

(B)

(C)

(D)

Answer: (D) 

 

Q. 8 Seven friends O, P, Q, R, S, T and U are watching a movie sitting in a row. P is sitting at one extreme end. Q is sitting to the immediate left of S. P is sitting second to the right of T. U is not sitting at any extreme end. O is sitting between R and T. Who is sitting at the extreme left end? 

(A)

(B)

(C)

(D)

Answer: (B) 

 

Q. 9 Select the option that is related to the third term in the same way as the second term is related to the first term. 

Ichthyology : Fishes :: Hematology : ? 

(A) Ligaments 

(B) Blood 

(C) Tissue 

(D) Bone 

Answer: (B) 

Explanation: 

Ichthyology is the branch of zoology that deals with fishes while Hematology is the branch of medicine involving study and treatment of the blood. 

=> Ans – (B) 

 

Q. 10 Select the option that will correctly replace the question mark (?) in the series. 

5, 17, 34, 61, ? 

(A) 105 

(B) 103 

(C) 115 

(D) 125 

Answer: (B) 

Explanation: 

The pattern followed is that numbers with successive multiples of 5 between them are added. 

5 + 12 = 17 

17 + 17 = 34 

34 + 27 = 61 

61 + 42 = 103 

=> Ans – (B) 

 

Q. 11 Select the Venn diagram that best represents the given set of classes. 

Paragraph, Word, Sentence 

Answer: (B) 

 

Q. 12 Choose the option that would follow next in the given figure series. 

Answer: (B) 

Explanation: 

The pattern followed is that the little gap between the two triangles in the first figure is pointing at top left and is moving in anti clockwise direction, i.e. in the second figure is pointing at left side, then bottom left and then at bottom, thus in the missing figure, it will face bottom right as shown in second figure. 

=> Ans – (B) 

 

Q. 13 Find out the two signs to be interchanged to make the following equation correct. 

25 + 25 × 25 − 25 ÷ 25 = −599 

(A) + and × 

(B) × and ÷ 

(C) − and × 

(D) − and ÷ 

Answer: (B) 

Explanation: 

Expression : 

25 + 25 × 25 − 25 ÷ 25 = −599 

+ × 

(A) : and 

L.H.S. 

≡ 25 × 25 + 25 − 25 ÷ 25 

625 + 25 − 1 = 649 = 

× ÷ 

(B) : and 

L.H.S. 

≡ 25 + 25 ÷ 25 − 25 × 25 

25 + 1 − 625 = −599 = 

= R.H.S. 

=> Ans – (B) 

 

Q. 14 Choose the option that most closely resembles the mirror image of the given figure. 

Answer: (A) 

Explanation: 

The last image is same as the question figure, hence it is eliminated, moreover the shape of the ring will remain vertical, hence the 

second and third options are also not possible. Only the position of stars will be changed as shown in first figure. => Ans – (A) 

 

Q. 15 Choose the odd number out of the given options. 

(A) 65 

(B) 52 

(C) 26 

(D) 97 

Answer: (D) 

Explanation: 

Among the given numbers, except for 97, all the numbers are divided by 13. Also, 97 is the only prime number given. => Ans – (D) 

 

Q. 16 Select the option that will correctly replace the question mark (?) in the series. 

102, 98, 82, 46, ? 

(A) 20 

(B) 18 

(C) -20 

(D) -18 

Answer: (D) 

Explanation: 

Squares of consecutive even natural numbers are subtracted. 

(2)2 

102 – = 98 

(4)2 

98 – = 82 

(6)2 

82 – = 46 

(8)2 

46 – = -18 

=> Ans – (D) 

 

Q. 17 Select the option that is related to the third term in the same way as the second term is related to the first term. 

Famous : Renowned : Ban : ? 

(A) Use 

(B) Packet 

(C) Allow 

(D) Prohibition 

Answer: (D) 

Explanation: 

Expression = Famous : Renowned : Ban : ? 

Famous and renowned are synonyms, similarly synonym of ban is prohibition

=> Ans – (D) 

 

Q. 18 The statements below are followed by two conclusions labeled I and II. Assuming that the information in the statements is true, even if it appears at variance with generally established facts, decide which conclusion(s) logically and definitely follow(s) from the information given in the statements. 

Statements: 

1) Some manure are ammonia. 

2) Some ammonia are potash. 

Conclusion: 

I. Some ammonia are manure. 

II. Some potash are ammonia. 

(A) Only conclusion I follows. 

(B) Both conclusions follow. 

(C) Neither conclusion I nor conclusion II follows. 

(D) Only conclusion II follows. 

Answer: (B) 

 

Q. 19 Select the option that is related to the third term in the same way as the second term is related to the first term. 

LEVEL : MFVDK :: FENCE 😕 

(A) GFND(B) 

(B) FGND(B) 

(C) GFNB(D) 

(D) FGNB(D) 

Answer: (C) 

 

Q. 20 In a certain code, ABOVE is written as ABEOV. How will QUESTION be written in that code? 

(A) EINOQSTU 

(B) EINQOSTU 

(C) EINOQTSU 

(D) USINVSIN 

Answer: (A) 

Explanation: 

ABOVE is written as ABEOV. 

The pattern followed is that the letters are arranged in increasing order as per English alphabetical order. Thus, QUESTION : EINOQSTU 

=> Ans – (A) 

 

Q. 21 Select the option that will correctly replace the question mark (?) in the given pattern. 

(A) 13 

(B)

(C) 50 

(D) 49 

Answer: (C) 

Explanation: 

If we start from 5 and move anti clockwise, we get the pattern that consecutive odd numbers are added. 

5 + 5 = 10 

10 + 7 = 17 

17 + 9 = 26 

26 + 11 = 37 

37 + 13 = 50 

=> Ans – (C) 

 

Q. 22 J, K L, M, N and are six teachers. Each one teaches a different subject out of Hindi, English, Math, Science, Social Science and Arts, not necessarily in the same order. Each of them teaches on only one day, from Monday to Saturday. not necessarily in the same order. 

J teaches Science on Wednesday. O teaches Math second after J. K teaches on the first day of the week, but teaches neither Hindi nor English. M teaches English before N and L teaches Arts before J. Which subject is taught on Saturday? 

(A) Hindi 

(B) English 

(C) Social Science 

(D) Science 

Answer: (A) 

 

Q. 23 Choose the odd one out of the given options. 

(A) Ear 

(B) Kidney 

(C) Eye 

(D) Nose 

Answer: (B) 

Explanation: 

The organs ear, eyes and nose can be seen from outside and can be touched, but kidney being inside the human body is the odd one. => Ans – (B) 

 

Q. 24 In a certain code. DIG is written as 20. How will FEED be written in that code? 

(A) 80 

(B) 60 

(C) 20 

(D) 90 

Answer: (C) 

Explanation: 

If all the alphabets in the English alphabetical order are written sequentially, then A=1, B=2, C=3, D=4 and so on. ≡ 4 + 9 + 7 = 20 

Then, DIG 

≡ 6 + 5 + 5 + 4 = 20 

Similarly, FEED

=> Ans – (C) 

 

Q. 25 Select the option that will correctly replace the question mark (?) in the series. 

QFM, OHL, MJK, ? 

(A) KJL 

(B) KLJ 

(C) JKL 

(D) LKJ 

Answer: (B) 

Explanation: 

Series : QFM, OHL, MJK, ? 

The pattern followed in each letter of the terms is : 

1st letter : Q (-2 letters) = O (-2 letters) = M (-2 letters) = K 

2nd letter : F (+2 letters) = H (+2 letters) = J (+2 letters) = L 

3rd letter : M (-1 letter) = L (-1 letter) = K (-1 letter) = J 

Thus, missing term = KLJ 

=> Ans – (B) 

General knowledge 

Instructions 

For the following questions answer them individually 

 

Q. 26 The Himalayas are an example of ……….. mountains. 

(A) block 

(B) plateau 

(C) volcanic 

(D) fold 

Answer: (D) 

 

Q. 27 The raga ‘Miya Malhar’ was created by: 

(A) Tansen 

(B) Mira Bai 

(C) Amir Khusro 

(D) Pandit Jasraj 

Answer: (A) 

 

Q. 28 Pokkalli, a unique variety of rice which can sustain in salinity is endemic to which state of India? 

(A) Goa 

(B) Andhra Pradesh 

(C) Kerala 

(D) Telangana 

Answer: (C) 

 

Q. 29 With reference to the Kangra School of Painting, which of the following statements is correct? 

(A) Its theme is prosaic in nature 

(B) It uses strong colours with deep Red and Black 

(C) It was inspired by the Vaishnavite traditions 

(D) It was developed during the reign of Samudragupta 

Answer: (C) 

 

Q. 30 When the government’s total revenue, excluding borrowings, is less than the planned expenditure,it is called: 

(A) revenue deficit 

(B) budget shortfall 

(C) fiscal deficit 

(D) deficit financing 

Answer: (C) 

 

Q. 31 “Sanitation is more important than independence” — Who said this? 

(A) C Rajagopalachari 

(B) Mahatma Gandhi 

(C) Jawaharlal Nehru 

(D) Subhash Chandra Bose 

Answer: (B) 

 

Q. 32 “Sattriya” is a dance form which has its origins in which state of India? 

(A) Karnataka 

(B) Andhra Pradesh 

(C) Tamil Nadu 

(D) Assam 

Answer: (D) 

 

Q. 33 With which sport is Durand Cup associated? 

(A) Football 

(B) Hockey 

(C) Tennis 

(D) Cricket 

Answer: (A) 

 

Q. 34 Ozone depletion is greatest near: 

(A) tropics 

(B) equator 

(C) poles 

(D) oceans 

Answer: (C) 

 

Q. 35 Which institution is responsible for calculating the GDP of India? 

(A) Ministry of Finance 

(B) Reserve Bank of India 

(C) Central Statistics Office 

(D) Chief Economic Advisor 

Answer: (C) 

 

Q. 36 When you heat a China dish containing 1gm of Copper powder, a Black substance is formed on the surface of the powder due to: 

(A) Atomization 

(B) Carbonization 

(C) Oxidation 

(D) Desalinization 

Answer: (C) 

 

Q. 37 Bedaquiline drug was developed to treat which of the following diseases? 

(A) Acute Myeloid Leukemia 

(B) Cholera 

(C) MDR-TB 

(D) Osteoarthritis 

Answer: (C) 

 

Q. 38 Which of the following features are used by Protista organisms for locomotion? 

(A) Maxillia 

(B) Flagella 

(C) Ephemera 

(D) Chimera 

Answer: (B) 

 

Q. 39 Andal, a Bhakti saint was associated to which of the following religious sects? 

(A) Vaishnavism 

(B) Shaivism 

(C) Shaktism 

(D) Buddhism 

Answer: (A) 

 

Q. 40 When were the first modern Olympic Games held? 

(A) 1896 

(B) 1935 

(C) 1943 

(D) 1921 

Answer: (A) 

 

Q. 41 Which of the following rivers flows into the Arabian Sea? 

(A) Narmada 

(B) Kaveri 

(C) Krishna 

(D) Godavari 

Answer: (A) 

 

Q. 42 Which is the highest peace time gallantry award in India? 

(A) Ashok Chakra 

(B) Kirti Chakra 

(C) Mahavir Chakra 

(D) Param Vir Chakra 

Answer: (A) 

 

Q. 43 Who is the author of the book ‘The Discovery of India’? 

(A) Atal Bihari Vajpayee 

(B) Indira Gandhi 

(C) Mahatma Gandhi 

(D) Jawaharlal Nehru 

Answer: (D) 

 

Q. 44 The Planning Commission was set up by the Government of India in the year: 

(A) 1948 

(B) 1950 

(C) 1949 

(D) 1951 

Answer: (B) 

 

Q. 45 Which of the following is NOT a feature of the Constitution of India? 

(A) Separation of Powers 

(B) Unitary System 

(C) Fundamental Rights 

(D) Federalism 

Answer: (B) 

 

Q. 46 As per 2011 census, which of the following states in India has the lowest population density? 

(A) Sikkim 

(B) Chhattisgarh 

(C) Arunachal Pradesh 

(D) Himachal Pradesh 

Answer: (C) 

 

Q. 47 A loan becomes a Non-Performing Asset (NPA) when the interest or principal becomes overdue for a period of: 

(A) 5 years 

(B) 90 days 

(C) 180 days 

(D) 365 days 

Answer: (B) 

 

Q. 48 A Eukaryotic cell has membrane bound organelles, including a nucleus, which allow cellular processes to be carried out in ……… 

(A) Bifurcation 

(B) Isolation 

(C) Inclusion 

(D) Cassation 

Answer: (B) 

 

Q. 49 The ‘Gandhara School of Art’ was developed under the ……… Dynasty. 

(A) Kushana 

(B) Maurya 

(C) Nanda 

(D) Chola 

Answer: (A) 

 

Q. 50 The Constituent Assembly met for the first time in ……….. 

(A) 1945 

(B) 1946 

(C) 1947 

(D) 1948 

Answer: (B) 

Quant 

Instructions 

For the following questions answer them individually 

Q. 51 Three persons A, B and C started a business with their shares in the ratio 3 : 4: 5. After 4 months B withdrew his 50% share and C withdrew his 20% share 4 months prior to completion of the year. If total profit in the year is ₹ 31,000 then find the share of A in the profit. 

(A) ₹7750 

(B) ₹9000 

(C) ₹9300 

(D) ₹10330 

Answer: (B) 

 

Q. 52 Marked price of an article is ₹ 8840. If discount of 35% is given on the article, then what will be the selling price of the article? 

(A) ₹6324 

(B) ₹6176 

(C) ₹6548 

(D) ₹5746 

Answer: (D) 

 

Q. 53 Anil, Dinesh and Deepak together can complete a work in 40 days. Anil and Dinesh together can complete the same work in 50 days. In how many days Deepak alone can complete the same work? 

(A) 90 days 

(B) 200 days 

(C) 150 days 

(D) 75 days 

Answer: (B) 

 

Q. 54 Suraj covers a distance of 18 km in 12 minutes. If his speed is decreases by 30 km/hr, then how much time will Suraj take to cover the same distance? 

(A) 12 minutes 

(B) 16 minutes 

(C) 20 minutes 

(D) 18 minutes 

Answer: (D) 

 

Q. 55 The difference between the simple interest and compound interest(interest is compounded half yearly) on a sum at the rate of 25% per annum for one year is ₹ 4375. What will be the principal? 

(A) ₹ 280000 

(B) ₹ 85000 

(C) ₹ 80000 

(D) ₹ 75000 

Answer: (A) 

 

Q. 56 A shopkeeper sells two articles, one of them with a profit of 10% for ₹ 1100 and another with a loss of 10% for ₹ 900. What will be the profit or loss percent? 

(A) 10% profit 

(B) Neither profit nor loss 

(C) 20% profit 

(D) 20% loss 

Answer: (B) 

 

Q. 57 A bag has ₹ 785 in the denomination of ₹ 2, ₹ 5 and ₹ 10 coins. The coins are in the ratio of 6 : 9 : 10. How many coins of ₹ 5 are in the bag? 

(A) 60 

(B) 12 

(C) 45 

(D) 24 

Answer: (C) 

 

Q. 58 The table given below shows the number of students enrolled in different courses of different colleges. 

Which college has the highest average number of students per course? 

(A) C4 

(B) C2 

(C) C3 

(D) C5 

Answer: (C) 

Explanation: 

Total number of students per course in : 

C4 : 76 + 52 + 75 + 120 = 323 

C2 : 68 + 58 + 71 + 112 = 309 

C3 : 72 + 54 + 88 + 125 = 339 [max] 

C5 : 69 + 60 + 79 + 126 = 334 

=> Ans – (C) 

 

Q. 59 In the table given below Marks of 4 students in 5 different Subjects are given. Total Marks of each subject is 100. 

What is the difference in average marks of students S1 and S4? 

(A) 1.4 

(B) 3.4 

(C)

(D) 2.4 

Answer: (D) 

 

Q. 60 If 30% of x is the same as 40% of y then what is y : x ? 

(A) 4 : 3 

(B) 3 : 4 

(C) 7 : 4 

(D) 4 : 7 

Answer: (B) 

 

Q. 61 Three cubes each of sides 7 cm are joined end to end. Find the surface area of the resulting solid. 

(A) 686 cm2 

(B) 866 cm 

(C) 882 cm2 

(D) 343 cm 

Answer: (A) 

 

Q. 62 If a 32 year old man is replaced by a new man,then the average age of 42 men increases by 1 year. What is the age of the new man? 

(A) 72 years 

(B) 75 years 

(C) 74 years 

(D) 73 years 

Answer: (C) 

 

Q. 63 A solid toy is in the form of hemisphere surmounted by right circular cone. Height of the cone is 4 cm and diameter of the base is 6 cm. What will be the surface area of the toy? 

(A) 660/7 cm 

(B) 528/7 cm 

(C) 726/7 cm 

(D) 66/7 cm 

Answer: (C) 

 

Q. 64 What is the value of [100 ÷ 50 − 25 − 55 of (⅓ × 11/9 )] ? 

(A) -1156/27

(B) -1382/27

(C) -1321/27

(D) -1226/27

Answer: (D) 

 

Q. 65 From solid cylinder with base diameter 14 cm and height 20 cm hemispherical parts were cut from both the ends of the cylinder. Find the volume of the resultant object? 

(A) 4312/3 cm3

(B) 4928/3 cm3

(C) 9240/3 cm3

(D) 13552/3 cm3

Answer: (B) 

 

Q. 66 A sum of ₹ 4000 is lent on simple interest at the rate of 10% per annum. Simple interest for 5 years is how much more than the simple interest for 3 years? 

(A) ₹ 400 

(B) ₹ 600 

(C) ₹ 800 

(D) ₹ 1200 

Answer: (C) 

 

Q. 67 14 observations are 10, 11, 11, 12, 13, 14, 12, 13, 10, 12, 12, 14, 14 and 15. What is the median of the given observations? 

(A) 12 

(B) 13 

(C) 14 

(D) 10 

Answer: (A) 

 

Q. 68 If Vijay is travelling at the speed of 17.5 m/s, then how much distance will Vijay cover in 26 hours? 

(A) 1474 km 

(B) 1562 km 

(C) 1572 km 

(D) 1638 km 

Answer: (D) 

 

Q. 69 The table given below shows the number of students enrolled in different courses of different colleges. 

What is the average number of students per college enrolled in course B2? 

(A) 60 

(B) 56 

(C) 58 

(D) 53 

Answer: (B) 

 

Q. 70 Product of Highest Common Factor and Least Common Multiple of two numbers is 216. If one of the numberis 12, then what will be the ratio of these two numbers? 

(A) 3 : 5 

(B) 1 : 6 

(C) 2 : 3 

(D) 1 : 2 

Answer: (C) 

 

Q. 71 What is the value of 28 ÷ (11 − 4) + 384 ÷ (15 + 16 ÷ 4 + 4) ? 

(A) 472/23 

(B) 476/23 

(C) 512/23 

(D) 524/23 

Answer: (B) 

 

Q. 72 If 40% of a number is 105 more than the one-fourth of that number, then what will be the difference between the number and 35% of the number? 

(A) 315 

(B) 495 

(C) 455 

(D) 375 

Answer: (C) 

 

Q. 73 P alone can complete a work in 16 days and Q alone can complete the same work in 20 days. P and Q start the work together but Q leaves the work 7 days before the completion of work. In how many days the total work will be completed? 

(A) 9 days 

(B) 14 days 

(C) 12 days 

(D) 18 days 

Answer: (C) 

 

Q. 74 The average monthly income of P and Q is ₹ 20000. The average monthly income of Q and R is ₹ 40000. Monthly income of R is how much more than monthly income of P? 

(A) ₹40000 

(B) ₹80000 

(C) ₹20000 

(D) ₹25000 

Answer: (A) 

 

Q. 75 Raju purchased glasses at the rate of ₹ 15 per glass and sold them at the rate of ₹ 12 per glass. On selling 42 glasses, what will be the loss percentage? 

(A) 33.33% 

(B) 25% 

(C) 20% 

(D) 14.28% 

Answer: (C) 

English 

Instructions 

For the following questions answer them individually 

Q. 76 Select the option that means the same as the given idiom. 

To miss the boat 

(A) To miss an opportunity 

(B) To miss the journey by boat 

(C) To fail to complete the work 

(D) To decide to travel by air 

Answer: (A) 

 

Q. 77 Select the antonym of the given word 

CONTRARY 

(A) Predetermined 

(B) Agreeable 

(C) Balanced 

(D) Neutral 

Answer: (B) 

 

Q. 78 Select the most appropriate word to fill in the blank. 

India’s Republic Day Parade had, among the main …………., an all-women’s marching contingent. 

(A) displays 

(B) spectacle 

(C) demonstrations 

(D) attractions 

Answer: (D) 

 

Q. 79 From the given options, identify the segment in the sentence which contains the grammatical error. Having worked for the whole day, you shall have taken some rest and begun again. 

(A) Having worked for 

(B) some rest and begun again 

(C) the whole day 

(D) you should have taken 

Answer: (D) 

 

Q. 80 Select the most appropriate option to substitute the underlined segment in the given sentence. If there is no need to substitute it, select No improvement. 

In the interview held recently, I answer the questions as best as I could. 

(A) I answer the question as bestas I could. 

(B) I answered the questions as best as I could. 

(C) I answered the question as best as Lean. 

(D) No improvement. 

Answer: (B) 

 

Q. 81 From the given options, identify the segment in the sentence which contains the grammatical error. The researchers have found that all species makes a constructive contribution to the ecology of the earth. 

(A) a constructive contribution to 

(B) that all species makes 

(C) the ecology of the earth. 

(D) The researchers have found 

Answer: (B) 

 

Q. 82 Select the word which means the same as the group of words given. 

The method and practice of teaching 

(A) Psychology 

(B) Methodology 

(C) Pedagogy 

(D) Philosophy 

Answer: (C) 

 

Q. 83 Select the most appropriate option to substitute the underlined segment in the given sentence.If there is no need to substitute it, select No improvement. 

As you can see from my resume, I am working in Mumbai since 2016. 

(A) No improvement. 

(B) I am work in Mumbai since 2016. 

(C) I am working in Mumbai from 2016. 

(D) I have been working in Mumbai since 2016. 

Answer: (D) 

 

Q. 84 From the given options, identify the segment in the sentence which contains the grammatical error. One could argue that these are the compulsions of parliamentary democracy for smooth functioning for the State. 

(A) these are the compulsions 

(B) of parliamentary democracy 

(C) functioning for the State. 

(D) One could argue that 

Answer: (C) 

 

Q. 85 Select the word which means the same as the group of words given. 

(A) person who presents a radio/television programme 

(A) Alchemist 

(B) Reporter 

(C) Anarchist 

(D) Anchor 

Answer: (D) 

Instructions 

In the following passage some words have been deleted. Fill in the blanks with the help of the alternatives given. 

Passage: 

Given That the availability of safe drinking water touches the lives of everyone, the ………(1) of Safe Drinking Water Mission can hardly be underestimated. Undoubtedly, this is one of the most ………(2) public goods problems to solve. The private solution of reaching out for bottled water due to risks posed by tap water is a ………….(3) one. Even the poor are forced to choose between risking hospital expenses and buying bottled water. Taking into consideration the ………..(4) scenario, it is the duty of every citizen to find out possible and inexpensive means to ………….(5) that safe drinking water is available to all. 

 

Q. 86 Select the most appropriate option that will fill in the blank number 1. 

(A) significance 

(B) safety 

(C) progress 

(D) consequence 

Answer: (A) 

 

Q. 87 Select the most appropriate option that will fill in the blank number 2. 

(A) difficult 

(B) complete 

(C) whole 

(D) ready 

Answer: (A) 

 

Q. 88 Select the most appropriate option that will fill in the blank number 3. 

(A) simple 

(B) material 

(C) factual 

(D) costly 

Answer: (D) 

 

Q. 89 Select the most appropriate option that will fill in the blank number 4. 

(A) public 

(B) present 

(C) general 

(D) rural 

Answer: (B) 

 

Q. 90 Select the most appropriate option that will fill in the blank number 5. 

(A) ensure 

(B) endorse 

(C) confirm 

(D) assure 

Answer: (A) 

Instructions 

For the following questions answer them individually 

 

Q. 91 Select the correctly spelt word. 

(A) Buraucracy 

(B) Bureaucrasy 

(C) Bureucracy 

(D) Bureaucracy 

Answer: (D) 

 

Q. 92 Select the synonym of the given word. 

MITIGATE 

(A) Improve 

(B) Excuse 

(C) Lessen 

(D) Moderate 

Answer: (C) 

 

Q. 93 Select the synonym of the given word. 

INDUSTRIOUS 

(A) Prompt 

(B) Fruitful 

(C) Diligent 

(D) Practical 

Answer: (C) 

 

Q. 94 Select the most appropriate option to substitute the underlined segment in the given sentence. If there is no need to substitute it, select No improvement. 

The river is flooded and it has overflown its banks. 

(A) it overflowing its banks. 

(B) it has overflow its banks. 

(C) No improvement. 

(D) it overflow its banks. 

Answer: (C) 

 

Q. 95 Select the most appropriate option to fill in the blank. 

We need not worry about anything ……….. his commitment to the work he is assigned. 

(A) access 

(B) accept 

(C) expect 

(D) except 

Answer: (D) 

 

Q. 96 Select the antonym of the given word 

SERENE 

(A) Splendid 

(B) Superb 

(C) Showy 

(D) Stressed 

Answer: (D) 

 

Q. 97 Select the most appropriate option to fill in the blank. 

If an individual keeps his fingers crossed he ……… for the best. 

(A) reacts 

(B) suspects 

(C) hopes 

(D) responds 

Answer: (C) 

 

Q. 98 Select the correctly spelt word. 

(A) Mathmetics 

(B) Mathamatics 

(C) Mathametics 

(D) Mathematics 

Answer: (D) 

 

Q. 99 Select the most appropriate option to fill in the blank. 

Sometimes it is necessary for an author to know what is going on in the minds of his ……….. 

(A) listeners 

(B) viewers 

(C) audience 

(D) characters 

Answer: (D) 

 

Q. 100 Select the option that means the same as the given idiom. 

To set aside 

(A) To move side ways 

(B) to reject an offer 

(C) To stop temporarily 

(D) To finish a work 

Answer: (C) 

SSC GD 18 Feb 2019 Shift-I Previous Year Paper

SSC GD 18th Feb 2019 Shift-I 

Reasoning 

Instructions 

For the following questions answer them individually 

Q. 1 Find out the two signs to be interchanged to make the following equation correct. 

10 + 10 × 10 − 10 ÷ 10 = −89 

(A) − and × 

(B) × and ÷ 

(C) + and × 

(D) − and ÷ 

Answer: (B) 

 

Q. 2 The statements below are followed by two conclusions labeled I and II. Assuming that the information in the statements is true, even if it appears at variance with generally established facts, decide which conclusion(s) logically and definitely follow(s) from the information given in the statements. 

Statements: 

1.) All owls are horses. 

2.) Some horses are hares. 

Conclusions: 

I. Some owls are hares. 

II. Some hares are horses. 

(A) Only conclusion II follows. 

(B) Neither conclusion I nor conclusion II follows. 

(C) Both conclusions follow. 

(D) Only conclusion I follows. 

Answer: (A) 

Explanation: 

The venn diagram for above statements is : 

Conclusions: 

I. Some owls are hares = may or may not be true 

II. Some hares are horses = true 

Thus, only conclusion II follows. 

=> Ans – (A) 

 

Q. 3 Select the option that will correctly replace the Q. mark (?) in the series. 

13, 25, 42, 69, ? 

(A) 111 

(B) 105 

(C) 95 

(D) 125 

Answer: (A) 

Explanation: 

The pattern followed is : 

13 + 12 = 25 

25 + 17 = 42 

42 + 27 = 69 

69 + 42 = 111 

The difference between the numbers is 5,10,15 respectively. 

=> Ans – (A) 

 

Q. 4 The statements below are followed by two conclusions labeled I and II. Assuming that the information in the statements is true, even if it appears at variance with generally established facts, decide which conclusion(s) logically and definitely follow(s) from the information given in the statements. 

Statements: 

1.) All pens are frogs. 

2.) Some crows are frogs. 

Conclusions: 

I. No pen is a crow. 

II. Some pens are crows. 

(A) Either conclusion I or conclusion II follows. 

(B) Only conclusion II follows. 

(C) Both conclusions follow. 

(D) Only conclusion I follows. 

Answer: (A) 

Explanation: 

The venn diagram for above statements is : 

Conclusions: 

I. No pen is a crow = may or may not be true 

II. Some pens are crows = may or may not be true 

Thus, either conclusion I or II follows. 

=> Ans – (A) 

 

Q. 5 Select the option that will correctly replace the Q. mark (?) in the series. 

112, 108, 92, 56, ? 

(A) -8 

(B) -7 

(C) -6 

(D)

Answer: (A) 

 

Q. 6 Choose the option in which the figure marked ‘X’ is embedded. (Rotation is not allowed) 

Answer: (C) 

Explanation: 

The Q. figure is embedded in following figure and is represented by red colour. 

=> Ans – (C) 

 

Q. 7 Choose the odd one out of the given options. 

(A) CEM 

(B) BHI 

(C) DFH 

(D) CFI 

Answer: (C) 

 

Q. 8 Select the option that will correctly replace the Q. mark (?) in the series.

SHO, QJN, OLM, ? 

(A) LWV 

(B) LWL 

(C) MNL 

(D) MML 

Answer: (C) 

Explanation: 

Expression : SHO, QJN, OLM, ? 

The pattern followed in each letter of the terms is : 

1st letter : S (-2) = Q (-2) = O (-2) = M 

2nd letter : H (+2) = J (+2) = (+2) = N 

3rd letter : O (-1) = N (-1) = M (-1) = L 

Thus, missing term : MNL 

=> Ans – (C) 

 

Q. 9 Select a figure from amongst the four alternatives that when placed in the blank space (?) of figure X will complete the pattern. (Rotation is not allowed). 

Answer: (D) 

Explanation: 

If we complete the above figure, then the missing pattern is represented by the red colour. 

=> Ans – (D) 

 

Q. 10 Select the option that is related to the third number in the same way as the second number is related to the first number. 

22 : 88 :: 37 : ? 

(A) 221 

(B) 144 

(C) 169 

(D) 370 

Answer: (D) 

Explanation: 

The number on the left is multiplied by the sum of its digits. 

Eg :- 

22 × (2 + 2) = 22 × 4 = 88 

Similarly, 

37 × (3 + 7) = 37 × 10 = 370 

=> Ans – (D) 

 

Q. 11 Eight friends A, B, C, D, E, F, G and H are sitting around a circular table facing each other for a lunch. A is opposite D and third to the right of B. G is between A and F.H is to the right of A. E is between C and D. Who is sitting between B and E? 

(A)

(B)

(C)

(D) (C) 

Answer: (A) 

 

Q. 12 Seven friends O, P, Q, R, S, T and U are watching a movie sitting in a row.P is sitting at one extreme end.Q is sitting to the immediate left of S. P is sitting second to the right of T. U is not sitting at any extreme end and is sitting between R and T. Who is sitting between Q and R? 

(A)

(B)

(C)

(D)

Answer: (D) 

 

Q. 13 Select the option that is related to the third term in the same way as the second term is related to the first term. 

Ichthyology : Fishes :: Pedology : ? 

(A) Shells 

(B) Names 

(C) Soil 

(D) Moon 

Answer: (C) 

Explanation: 

Ichthyology is a branch of zoology that deals with fishes, similarly Pedology means the study of soils. => Ans – (C) 

 

Q. 14 Select the option that will correctly replace the Q. mark (?) in the given pattern. 

(A) 343 

(B)

(C) 37 

(D) 49 

Answer: (A) 

 

Q. 15 Choose the option that would follow next in the given figure series. 

Answer: (C) 

Explanation: 

In the given figure, the little gap between the two triangles is first pointing at top left, and then is moving in clockwise direction. Thus in 

the next figure, it is facing top, then top right followed by right direction. 

Thus, in the next figure, it will point towards bottom right as shown in third figure. 

=> Ans – (C) 

 

Q. 16 A square transparent sheet with a pattern is given. How will the pattern appear when the transparent sheet is folded along the dotted line? 

Answer: (D) 

 

Q. 17 J, K L, M, N and O are six teachers. Each one teaches a different subject out of Hindi, English, Math, Science, Social Science and Arts, not necessarily in the same order. Each of them teaches on only one day, from Monday to Saturday, not necessarily in the same order. 

J teaches Science on Wednesday. O teaches Math second after J. K teaches on the first day of the week, but teaches neither Hindi nor English. M teaches English before N and L teaches Arts before J. Who teaches on Tuesday? 

(A)

(B)

(C)

(D)

Answer: (C) 

 

Q. 18 Choose the option that most closely resembles the mirror image of the given figure when mirror is placed at the right side. 

Answer: (A) 

Explanation: 

When a mirror is placed at the right side, the shape of the circle will not change, hence the last two options are eliminated. Also, the position of the stars will be shifted towards right side, hence first option is the correct image. => Ans – (A) 

 

Q. 19 In a certain code, POTATO is written as AOOPTT. How will BARBER be written in that code? 

(A) ABBERR 

(B) ABBRER 

(C) ARBERB 

(D) ABRBER 

Answer: (A) 

Explanation: 

POTATO is written as AOOPTT. 

The pattern followed is that the letters are arranged in ascending order according to the English alphabetical order. Hence BARBER is written as ABBERR 

=> Ans – (A) 

 

Q. 20 Choose the odd number out of the given options. 

(A) 97 

(B) 89 

(C) 83 

(D) 63 

Answer: (D) 

Explanation: 

Except for 63, all are prime numbers, hence 63 is the odd one. 

=> Ans – (D) 

 

Q. 21 Select the Venn diagram that best represents the given set of classes. 

Wheat, Crops, Farmers 

Answer: (C) 

 

Q. 22 Choose the odd one out of the given options. 

(A) Yellow 

(B) Pink 

(C) Violet 

(D) Orange 

Answer: (B) 

Explanation: 

All except Pink are the colours seen in a rainbow, hence pink is the odd one. 

=> Ans – (B) 

 

Q. 23 Select the option that is related to the third term in the same way as the second term is related to the first term. 

Achieve : Fail :: Gloomy : ? 

(A) Chalk 

(B) Chant 

(C) Chase 

(D) Cheerful 

Answer: (D) 

Explanation: 

In the given pair, Fail is the opposite of Achieve, similarly antonym of gloomy is cheerful. 

=> Ans – (D) 

 

Q. 24 Select the option that is related to the third term in the same wayas the second term is related to the first term. 

BFH : DHJ :: GKM : ? 

(A) MKI 

(B) IMO 

(C) KMG 

(D) MKG 

Answer: (B) 

 

Q. 25 In a certain code, SAW is written as 38. How will FEED be written in that code? 

(A) 70 

(B) 45 

(C) 88 

(D) 30 

Answer: (C) 

Explanation: 

If the alphabets are of the English alphabetical order are written in reverse order and numbered sequentially, i.e. Z=1, Y=2, X=3, W=4 and so on. 

Thus, SAW = 8+26+4 = 38 

21 + 22 + 22 + 23 = 88 

Similarly, FEED = 

=> Ans – (C) 

General knowledge 

Instructions 

For the following questions answer them individually 

Q. 26 Which articles of the Indian Constitution cover the fundamental right against exploitation? 

(A) Articles 29-30 

(B) Articles 19-22 

(C) Articles 14-18 

(D) Articles 23-24 

Answer: (D) 

 

Q. 27 Where and against which country did Sachin Tendulkar score his 100th century? 

(A) Perth stadium, against West Indies 

(B) Shere Bangla stadium, against Bangladesh 

(C) Edenpark, against New Zealand 

(D) Sydney cricket ground, against Australia 

Answer: (B) 

 

Q. 28 In which state is Kaziranga National Park situated? 

(A) Assam 

(B) Madhya Pradesh 

(C) Rajasthan 

(D) Bihar 

Answer: (A) 

 

Q. 29 Which of these languages is declared ‘classical’ by the Government of India? 

(A) Marathi 

(B) Hindi 

(C) Odia 

(D) Pali 

Answer: (C) 

 

Q. 30 Arrange the following events in correct chronology: 

1) Champaran Satyagraha against indigo plantation 

2) Starting of non-cooperation Khilafat movement 

3) Rowlatt Satyagraha 

4) Arrival of Simon commission in India 

(A) 1-3-2-4 

(B) 3-4-2-1 

(C) 4-2-1-3 

(D) 1-2-3-4 

Answer: (A) 

 

Q. 31 What was the amendment done to the Indian Constitution by 52nd amendment done in 1985? 

(A) Anti-defection law was included 

(B) Amendment to the Union and State lists with respect to raising of taxes 

(C) The words ‘socialist secular’ added to the preamble 

(D) Formation of Sikkim as a state within the Indian Union 

Answer: (A) 

 

Q. 32 The Metturdam is built on which river and is in which state? 

(A) Tungabhadra river, Karnataka 

(B) Kali river, Karnataka 

(C) Kaveri river, Tamilnadu 

(D) Narmada river, Madhya Pradesh 

Answer: (C) 

 

Q. 33 Which of these personalities is popularly known as the ‘father of the Indian film industry’? 

(A) Amitabh Bachchan 

(B) Dadasaheb Phalke 

(C) Dev Anand 

(D) Rajesh Khanna 

Answer: (B) 

 

Q. 34 The silver coin introduced by Sher Shah Suri was called: 

(A) Tankah 

(B) Rupiya 

(C) Mohar 

(D) Dinar 

Answer: (B) 

 

Q. 35 The Kani Shaw! handicraft variety belongs to which of the following states of India? 

(A) Jammu & Kashmir 

(B) Arunachal Pradesh 

(C) Himachal Pradesh 

(D) Uttar Pradesh 

Answer: (A) 

 

Q. 36 Which of the following is not a term associated with credit? 

(A) Necessary documents 

(B) Collateral 

(C) Interest rate 

(D) Stamp by finance minister 

Answer: (D) 

 

Q. 37 Which of the following competitions is not associated with Indian football? 

(A) Ranji Trophy 

(B) Federation Cup 

(C) Santosh Trophy 

(D) Durand Cup 

Answer: (A) 

 

Q. 38 Scurvy is caused by the deficiency of which vitamin? 

(A) Vitamin C 

(B) Vitamin A 

(C) Vitamin D 

(D) Vitamin K 

Answer: (A) 

 

Q. 39 In which Indian state is bamboo drip irrigation system a very old practice? 

(A) Chattisgarh 

(B) Meghalaya 

(C) Telangana 

(D) Maharashtra 

Answer: (B) 

 

Q. 40 Which is the highest quality hard coal of the below? 

(A) Peat 

(B) Lignite 

(C) Bituminous 

(D) Anthracite 

Answer: (D) 

 

Q. 41 The Treaty of Purandar was signed between: 

(A) Afghans and Portuguese 

(B) Nawab of Bengal and Rajputs 

(C) Eastern Gangas and Cholas 

(D) Mughals and Marathas 

Answer: (D) 

 

Q. 42 Which of the motions is passed in the parliament to remove the President of India? 

(A) Impeachment Motion 

(B) Censure Motion 

(C) Cut Motion 

(D) No Confidence Motion 

Answer: (A) 

 

Q. 43 Which of these is an organisation that promotes sustainable management and conservation of tropical forests? 

(A) United Nations Forum on Forests 

(B) Greenpeace 

(C) World Wide Fund for Nature 

(D) International Tropical Timber Organization 

Answer: (D) 

 

Q. 44 Which of the following issues currency notes in India? 

(A) PNB 

(B) RBI 

(C) SBI 

(D) ICICI 

Answer: (B) 

 

Q. 45 What is the neutral value of the pH scale? 

(A) Equal to 14 

(B) Less than 7 

(C) Equal to 7 

(D) Above 7 

Answer: (C) 

 

Q. 46 Bananas turn brown in color because: 

(A) an enzyme converts starch into sugar 

(B) the air reacts with the enzymes in the banana 

(C) Bisphosphonates reacts with the enzymes in the banana 

(D) an enzyme converts sugarinto starch 

Answer: (B) 

 

Q. 47 On which of the below can an excise duty charged? 

(A) A company’s revenue 

(B) Export of goods and services 

(C) Import of goods and services 

(D) Goods manufactured 

Answer: (D) 

 

Q. 48 Who is the author of the book ‘India’s Export Trends and Prospects for Self-Sustained Growth’? 

(A) Shashi Tharoor 

(B) Manmohan Singh 

(C) Atal Bihari Vajpayee 

(D) Narendra Modi 

Answer: (B) 

 

Q. 49 In which Indian state is the Hornbill festival celebrated? 

(A) Nagaland 

(B) Manipur 

(C) Sikkim 

(D) Tripura 

Answer: (A) 

 

Q. 50 What is the name of Malala Yousafzai’s biopic? 

(A) The Aviator 

(B) Frida 

(C) The social network 

(D) Gul Makai 

Answer: (D) 

Quant 

Instructions 

For the following questions answer them individually 

Q. 51 What is the median of the given data? 

2, 3, 2, 3, 6, 5, 4, 7 

(A)

(B) 4.5 

(C) 3.5 

(D)

Answer: (C) 

 

Q. 52 A sum becomes five times of itself in 8 years at simple interest. What is the rate of interest per annum? 

(A) 37.5% 

(B) 25% 

(C) 62.5% 

(D) 50% 

Answer: (D) 

 

Q. 53 The table given below shows the marks obtained by 4 students in 5 subjects. The maximum marks of each subject is 100.

What is the difference in the average marks per student for subjects S5 and S2? 

(A) 12 

(B) 15 

(C) 17.5 

(D) 19.5 

Answer: (B) 

 

Q. 54 The length breadth and height of a cuboid are in the ratio 4 : 3 : 2. If the volume of cuboid is 1536 cm, then what will be the total surface area of the cuboid? 

(A) 868 cm2 

(B) 416 cm2 

(C) 624 cm2 

(D) 832 cm

Answer: (D) 

 

Q. 55 The length, breadth and height of a solid cuboid is 20 cm, 16 cm and 12 cm respectively. If cuboid is melted to form identical cubes of side 4 cm, then what will be the number of identical cubes? 

(A) 56 

(B) 72 

(C) 90 

(D) 60 

Answer: (D) 

 

Q. 56 A bike covers a certain distance at a speed of 56 km/hr in 16 hours. How much time it will take to cover the same distance at the speed of 64 km/hr? 

(A) 14 hr 

(B) 18 hr 

(C) 16 hr 

(D) 12 hr 

Answer: (A) 

 

Q. 57 What is the value of (3 × 1500 ÷ 40 + 5 ÷ 2/7 of 70) ? 

(A) 433/4

(B) 435/4

(C) 451/4

(D) 473/4

Answer: (C) 

 

Q. 58 Curved surface area of a cylinder is 110 cm2. If the height of cylinder is 5 cm, then what will be the diameter of its base? 

(A) 14 cm 

(B) 7 cm 

(C) 3.5 cm 

(D) 10.5 cm 

Answer: (B) 

 

Q. 59 Mohit and Sumit start a business with investment of ₹ 74000 and ₹ 96000 respectively. If at the end of the year they earn profit in the ratio of 5 : 8, then what will be ratio of the time period for which they invest their money? 

(A) 37 : 32 

(B) 13 : 18 

(C) 25 : 32 

(D) 30 : 37 

Answer: (D) 

 

Q. 60 The value of discount of 18% on first article is equal to the value of discount of 13% on second article. What is the ratio of the marked price of two articles? 

(A) 17:24 

(B) 39:50 

(C) 13:18 

(D) 13:19 

Answer: (C) 

 

Q. 61 What will be the Highest Common Factor of 5, 10 and 20? 

(A)

(B) 10 

(C)

(D) 15 

Answer: (A) 

 

Q. 62 The average weight of 12 boxes is 63 kg. If four boxes having an average weight of 70 kg are removed, then what will be new average weight of the remaining boxes? 

(A) 60 kg 

(B) 59 kg 

(C) 60.5 kg 

(D) 59.5 kg 

Answer: (D) 

 

Q. 63 F alone can complete a work in 24 days and G alone can complete the same work in 32 days. F and G start the work together but G leaves the work 8 days before the completion of work. In how many days the total work will be completed? 

(A) 130/5 days 

(B) 106/7 days 

(C) 114/7 days 

(D) 120/7 days 

Answer: (D) 

 

Q. 64 Working 2 hours in a day, P can complete a work in 10 days and working 5 hours in a day, Q can complete the same work in 6 days. Working 3 hours in a day, in how many days both P and Q together can complete the same work? 

(A) 3 days 

(B) 6 days 

(C) 4 days 

(D) 5 days 

Answer: (C) 

 

Q. 65 Mohit purchases eggs at the rate of ₹ 3 per egg and sells them at the rate of ₹ 5 per egg. On selling 35 eggs, what will be the profit percentage? 

(A) 33.33% 

(B) 40% 

(C) 60% 

(D) 66.67% 

Answer: (D) 

 

Q. 66 A bag has white and yellow balls in the ratio 7 : 11 respectively. If the total number of balls is 108, then how many balls are yellow? 

(A) 66 

(B) 44 

(C) 64 

(D) 77 

Answer: (A) 

 

Q. 67 Varun purchases 75 articles for ₹ 3150 and sells them at a profit equal to the cost price of 15 articles. What will be the selling price of one article? 

(A) 42 

(B) 51.5 

(C) 50.4 

(D) 46.5 

Answer: (C) 

 

Q. 68 The average age of students is A and the average age of teachers is B. The number of the students is 45 times of the number of teachers. What is average age of students and teachers together? 

(A) 45A+8B/7 

(B) 45A+B/46 

(C) 45A+B/8 

(D) 45A+4B/23 

Answer: (B) 

 

Q. 69 The table given below shows the number of students enrolled in different courses of different colleges. 

What is the difference in the total number of students enrolled in colleges C3 and C4? 

(A) 13 

(B) 11 

(C) 12 

(D) 16 

Answer: (D) 

 

Q. 70 Ram, Sita and Salma invest ₹ 16000, ₹ 22000 and ₹ 18000 respectively to start a business. If the profit at the end of the year is ₹ 26600, then what is share of Ram? 

(A) ₹ 10450 

(B) ₹ 8550 

(C) ₹ 9650 

(D) ₹ 7600 

Answer: (D) 

 

Q. 71 A student got 24% marks in an exam and he failed by 56 marks. If he got 60% marks, then his marks are 70 more than the minimum passing marks. What is the maximum marks for the exam? 

(A) 350 

(B) 260 

(C) 380 

(D) 280 

Answer: (A) 

 

Q. 72 What is the value of [75 ÷ 25 of (4/3 − ½ + ⅙)]?

(A) ⅚ 

(B)

(C) 7/6 

(D)

Answer: (B) 

 

Q. 73 Raju travels at ¾ of his usual speed and reaches his office 24 minutes late than his usual time. If Raju travels at his usual speed, then what will be the time taken by him to reach his office? 

(A) 64 minutes 

(B) 72 minutes 

(C) 48 minutes 

(D) 56 minutes 

Answer: (B) 

 

Q. 74 Compound interest received on a sum in 5th year is ₹ 1200 (interest is compounded annually).If the rate of interest is 22.5% per annum, then what will be the interest in 6th year? 

(A) ₹ 1470 

(B) ₹ 1270 

(C) ₹ 1200 

(D) ₹ 270 

Answer: (A) 

 

Q. 75 The table given below shows the production of rice and the area under rice cultivation of a country for 5 consecutive years Y1, Y2, Y3, Y4 and Y5. 

What is the yield per square meter for the year Y3? 

(A) 37.66 kg 

(B) 63.33 kg 

(C) 45.33 kg 

(D) 52.50 kg 

Answer: (C) 

English 

Instructions 

For the following questions answer them individually 

Q. 76 Select the most appropriate word to fill in the blank. 

We should always remember not to ………. any problems and issues that may affect our career in any organisation. 

(A) affect 

(B) undertake 

(C) inflict 

(D) create 

Answer: (D) 

 

Q. 77 Select the most appropriate option to substitute the underlined segment in the given sentence.If there is no need to substitute it, select No Improvement. 

After you will return from Bangalore, I will come and meet you. 

(A) After you will be returning from 

(B) No improvement 

(C) After you returned from 

(D) After you return from 

Answer: (D) 

 

Q. 78 Select the antonym of the given word 

AWKWARD

(A) Problematic 

(B) Convenient 

(C) Difficult 

(D) Matchless 

Answer: (B) 

Instructions 

In the following passage some words have been deleted. Fill in the blanks with the help of the alternatives given. 

Passage: 

Seafood exports from India …………(1) yet to gather momentum as ……………(2) prices, adverse weather conditions and …………..(3) continue to affect stocking of shrimps, which account for about threefourths of the ………….(4) from export of marine products. This apart, seafood exporters are facing …………….(5) from the Seafood Import Monitoring Programme (SIMP) being implemented by the USandit is one of the largest buyers of Indian shrimps. 

 

Q. 79 Select the most appropriate option that will fill in the blank number 1. 

(A) be 

(B) has 

(C) are 

(D) was 

Answer: (C) 

 

Q. 80 Select the most appropriate option that will fill in the blank number 2. 

(A) low 

(B) little 

(C) small 

(D) high 

Answer: (D) 

 

Q. 81 Select the most appropriate option that will fill in the blank number 3. 

(A) diseases 

(B) viruses 

(C) bugs 

(D) maladies 

Answer: (A) 

 

Q. 82 Select the most appropriate option that will fill in the blank number 4. 

(A) takings 

(B) income 

(C) cash 

(D) revenue 

Answer: (D) 

 

Q. 83 Select the most appropriate option that will fill in the blank number 5. 

(A) rapture 

(B) pleasure 

(C) capture 

(D) pressure 

Answer: (D) 

Instructions 

For the following questions answer them individually 

 

Q. 84 From the given options, identify the segment in the sentence which contains the grammatical error. 

The government’s interim budget from this financial year contained several measures which have the forthcoming elections in mind. 

(A) from this financial year 

(B) Government’s interim budget 

(C) which have the 

(D) contained several measures 

Answer: (A) 

 

Q. 85 Select the most appropriate option to substitute the underlined segment in the given sentence.If there is no need to substitute it, select No Improvement. 

While learning to drive , one of the most of important things is to know the traffic rules first. 

(A) No improvement 

(B) When learned to drive 

(C) When learn driving 

(D) When learn to drive 

Answer: (A) 

 

Q. 86 From the given options, identify the segment in the sentence which contains the grammatical error. 

Soon after returning to office, the CBI chief transfer five officers. 

(A) returning to office, 

(B) the CBI chief 

(C) transfer five officers 

(D) Soon after 

Answer: (C) 

 

Q. 87 Select the correctly spelt word. 

(A) Substitushion 

(B) Substitution 

(C) Substituition 

(D) Sabstitution 

Answer: (B) 

 

Q. 88 Select the option that means the same as the given idiom. 

To cut both ends 

(A) To reject all arguments 

(B) To argue in support of both sides 

(C) To consider the truth and untruth 

(D) To argue against both sides of an issue 

Answer: (B) 

 

Q. 89 Select the antonym of the given word. 

EUPHORIA 

(A) Folklore 

(B) Lethargy 

(C) Song 

(D) Music 

Answer: (B) 

 

Q. 90 Select the correctly spelt word. 

(A) Magnanemous 

(B) Magnanomous 

(C) Magnanimous 

(D) Magnonimus 

Answer: (C) 

 

Q. 91 From the given options, identify the segment in the sentence which contains the grammatical error. The Builders’ Associations want GST exemption for government projects which have already proposed. 

(A) which have already proposed. 

(B) want GST exemption 

(C) for government projects 

(D) The Builders’ Associations 

Answer: (A) 

 

Q. 92 Select the most appropriate option to fill in the blank. 

Is it true that the students from the Space Research Centre ………. this evening? 

(A) are arriving 

(B) is arriving 

(C) has arriving 

(D) have arriving 

Answer: (A) 

 

Q. 93 Select the most appropriate option to substitute the underlined segment in the given sentence.If there is no need to substitute it, select No Improvement. 

These flowers smell sweetly and are sold at high prices in the market. 

(A) These flowers will smell sweetly 

(B) These flowers smelt sweetly 

(C) No improvement 

(D) These flowers smell sweet 

Answer: (D) 

 

Q. 94 Select the word which means the same as the group of words given. 

One who studies antique things 

(A) Historian 

(B) Sociologist 

(C) Archaeologist 

(D) Antiquarian 

Answer: (D) 

 

Q. 95 Select the synonym of the given word. 

INCORRIGIBLE 

(A) Reliable 

(B) Unwise 

(C) Intelligible 

(D) Unalterable 

Answer: (D) 

 

Q. 96 Select the most appropriate word to fill in the blank. 

Although I was ………….. of his new plans for growth of the company, I relied on him as he was very honest. 

(A) persuaded 

(B) committed 

(C) convinced 

(D) sceptical 

Answer: (D) 

 

Q. 97 Select the word which means the same as the group of words given. 

One who offers one’s services without anything in return 

(A) Volunteer 

(B) Entrepreneur 

(C) Businessman 

(D) Investor 

Answer: (A) 

 

Q. 98 Select the synonym of the given word. 

DISHEVELLED 

(A) Unsafe 

(B) Unclean 

(C) Uncertain 

(D) Untidy 

Answer: (D) 

 

Q. 99 Choose the correct meaning of the following idiom. 

To get away with 

(A) To escape from something 

(B) To go away from home 

(C) To finish an assignment 

(D) To go to an unknown place 

Answer: (A) 

 

Q. 100 Select the most appropriate option to fill in the blank. 

……………… example helps understand a concept better. 

(A) Few 

(B) (A) 

(C) An 

(D) One 

Answer: (C) 

×

Hello!

Click one of our representatives below to chat on WhatsApp or send us an email to info@vidhyarthidarpan.com

×